Vous êtes sur la page 1sur 75

MAT 235-02

October 15-19, 2019

1
CHAPTER 1

First Order Equations

Definition. A first order differential equation is called linear if it can be written as


y 0 + p(x)y = f (x). (1)
A first order differential equation that can’t be written like this is nonlinear . We say that
(1) is homogeneous if f ≡ 0; otherwise it’s nonhomogeneous. Since y ≡ 0 is obviously a
solution of the homgeneous equation
y 0 + p(x)y = 0,
we call it the trivial solution. Any other solution is nontrivial .
Example 1. The first order equations
x2 y 0 + 3y = x2 ,

xy 0 − 8x2 y = sin x,
xy 0 + (ln x)y = 0,
y 0 = x2 y − 2,
are not in the form (1), but they are linear, since they can be rewritten as
y 0 + 3/x2 y = 1,
y 0 − 8xy = sin x/x,
y 0 + ln x/xy = 0,
y 0 − x2 y = −2.
Example 2. Here are some nonlinear first order equations:
xy 0 + 3y 2 = 2x (because y is squared),
yy 0 = 3 (because of the product yy 0 ),
y 0 + xey = 12 (because of ey ).

Homogeneous Linear First Order Equations


We begin with the problem of finding the general solution of a homogeneous linear first
order equation. The nexamplet exampleample recalls a familiar result from calculus.
Example 3. Let a be a constant.
(1) Find the general solution of
y 0 − ay = 0. (2)
(2) Solve the initial value problem
y 0 − ay = 0, y(x0 ) = y0 .
Solution.
3
y

3.0

2.5
a=2

2.0
a = 1.5

1.5 a=1

1.0

a = −1
0.5
a = −2.5
a = −4
x
0.2 0.4 0.6 0.8 1.0

Figure 1. Solutions of y 0 − ay = 0, y(0) = 1

We already know from calculus that if c is any constant, then y = ceax satisfies (2).
However, let’s pretend we have forgotten this, and use this problem to illustrate a general
method for solving a homogeneous linear first order equation.
We know that (2) has the trivial solution y ≡ 0. Now suppose y is a nontrivial solution
of (2). Then, since a differentiable function must be continuous, there must be some open
interval I on which y has no zeros. We rewrite (2) as
y0
=a
y
for x in I. Integrating this shows that
ln |y| = ax + k, so |y| = ek eax ,
where k is an arbitrary constant. Since eax can never equal zero, y has no zeros, so y is
either always positive or always negative. Therefore we can rewrite y as
y = ceax (3)
where
ek if y > 0,

c=
−ek if y < 0.
This shows that every nontrivial solution of (2) is of the form y = ceax for some nonzero
constant c. Since setting c = 0 yields the trivial solution, all solutions of (2) are of the
form (3). Conversely, (3) is a solution of (2) for every choice of c, since differentiating
(3) yields y 0 = aceax = ay.
Imposing the initial condition y(x0 ) = y0 yields y0 = ceax0 , so c = y0 e−ax0 and

y = y0 e−ax0 eax = y0 ea(x−x0 ) .


Figure 1 show the graphs of this function with x0 = 0, y0 = 1, and various values of a.
··
^
4
y

c<0 c>0

c>0 c<0

Figure 2. Solutions of xy 0 + y = 0 on (0, ∞) and (−∞, 0)

Example 4. (1) Find the general solution of


xy 0 + y = 0. (4)
(2) Solve the initial value problem
xy 0 + y = 0, y(1) = 3. (5)
Solution.
We rewrite (4) as
1
y 0 + y = 0, (6)
x
where x is restricted to either (−∞, 0) or (0, ∞). If y is a nontrivial solution of (6), there
must be some open interval I on which y has no zeros. We can rewrite (6) as
y0 1
=−
y x
for x in I. Integrating shows that
ek
ln |y| = − ln |x| + k, so |y| = .
|x|
Since a function that satisfies the last equation can’t change sign on either (−∞, 0) or
(0, ∞), we can rewrite this result more simply as
c
y= (7)
x
where
ek if y > 0,

c=
−ek if y < 0.
We’ve now shown that every solution of (6) is given by (7) for some choice of c. (Even
though we assumed that y was nontrivial to derive (7), we can get the trivial solution by
5
setting c = 0 in (7).) Conversely, any function of the form (7) is a solution of (6), since
differentiating (7) yields
c
y0 = − 2 ,
x
and substituting this and (7) into (6) yields
1 c 1c
y0 + y = − 2 +
x x xx
c c
= − 2 + 2 = 0.
x x
Figure 2 shows the graphs of some solutions corresponding to various values of c
Imposing the initial condition y(1) = 3 in (7) yields c = 3. Therefore the solution of
(5) is
3
y= .
x
··
The interval of validity of this solution is (0, ∞). ^
Example 5.
The equation
y 0 = (x − y + 1)2
is neither separable nor linear. What can we do? How about trying to change variables,
so that in the new variables the equation is simpler. We use another variable v, which
we treat as a function of x. Let us try
v = x − y + 1.
We need to figure out y 0 in terms of v 0 , v and x. We differentiate (in x) to obtain
v 0 = 1 − y 0 . So y 0 = 1 − v 0 . We plug this into the equation to get
1 − v0 = v2.
In other words, v 0 = 1 − v 2 . Such an equation we know how to solve by separating
variables:
1
dv = dx.
1 − v2
So

1 v + 1
ln = x + C,
2 v − 1

v + 1 2x+2C
v − 1 = e ,

v+1
or = De2x for some constant D. Note that v = 1 and v = −1 are also solutions.
v−1
Now we need to unsubstitute to obtain
x−y+2
= De2x ,
x−y
6
and also the two solutions x − y + 1 = 1 or y = x, and x − y + 1 = −1 or y = x + 2. We
solve the first equation for y.
x − y + 2 = (x − y)De2x ,
x − y + 2 = Dxe2x − yDe2x ,
−y + yDe2x = Dxe2x − x − 2,
y (−1 + De2x ) = Dxe2x − x − 2,
Dxe2x − x − 2
y= .
De2x − 1
Note that D = 0 gives y = x + 2, but no value of D gives the solution y = x.
··
^
Remark 6. Substitution in differential equations is applied in much the same way
that it is applied in calculus. You guess. Several different substitutions might work.
There are some general patterns to look for. We summarize a few of these in a table.
When you see Try substituting
yy 0 v = y2
2 0
y y v = y3
(cos y)y 0 v = sin y
0
(sin y)y v = cos y
y 0 ey v = ey
Usually you try to substitute in the most complicated part of the equation with the
hopes of simplifying it. The above table is just a rule of thumb. You might have to
modify your guesses. If a substitution does not work (it does not make the equation any
simpler), try a different one.

Separable Equations
Definition 7. A first order differential equation is separable if it can be written as
h(y)y 0 = g(x), (8)
where the left side is a product of y 0 and a function of y and the right side is a function of
x. Rewriting a separable differential equation in this form is called separation of variables.
In the previous section, we used separation of variables to solve homogeneous linear
equations. In this section we’ll apply this method to nonlinear equations.
To see how to solve (8), let’s first assume that y is a solution. Let G(x) and H(y) be
antiderivatives of g(x) and h(y); that is,
H 0 (y) = h(y) and G0 (x) = g(x). (9)
Then, from the chain rule,
d
H(y(x)) = H 0 (y(x))y 0 (x) = h(y)y 0 (x).
dx
Therefore, (8) is equivalent to
d d
H(y(x)) = G(x).
dx dx
Integrating both sides of this equation and combining the constants of integration yields
H(y(x)) = G(x) + c. (10)
7
Although we derived this equation on the assumption that y is a solution of (8), we can
now view it differently: Any differentiable function y that satisfies (10) for some constant
c is a solution of (8). To see this, we differentiate both sides of (10), using the chain rule
on the left, to obtain
H 0 (y(x))y 0 (x) = G0 (x),
which is equivalent to
h(y(x))y 0 (x) = g(x)
because of (9).
In conclusion, to solve (8) it suffices to find functions G = G(x) and H = H(y) that
satisfy (9). Then any differentiable function y = y(x) that satisfies (10) is a solution of
(8).
Example 8. Solve the equation
y 0 = x(1 + y 2 ).
Solution.
Separating variables yields
y0
= x.
1 + y2
Integrating yields
x2
arctan y = +c
2
Thus,
x2
 
y = tan +c .
2
··
^
Example 9. (1) Solve the equation
x
y0 = − . (11)
y
(2) Solve the initial value problem
x
y 0 = − , y(1) = 1. (12)
y
(3) Solve the initial value problem
x
y 0 = − , y(1) = −2. (13)
y
Solution.
Separating variables in (11) yields
yy 0 = −x.
Integrating yields
y2 x2
= − + c, or, equivalently, x2 + y 2 = 2c.
2 2
The last equation shows that c must be positive if y is to be a solution of (11) on an open
interval. Therefore we let 2c = a2 (with a > 0) and rewrite the last equation as
x 2 + y 2 = a2 . (14)
8
This equation has two differentiable solutions for y in terms of x:

y= a2 − x2 , −a < x < a, (15)

and

y = − a2 − x 2 , −a < x < a. (16)
The solution curves defined by (15) are semicircles above the x-axis and those defined by
(16) are semicircles below the x-axis (Figure 3).
The solution of (12) is positive when x = 1; hence, it is of the form (15). Substituting
x = 1 and y = 1 into (14) to satisfy the initial condition yields a2 = 2; hence, the solution
of (12) is
√ √ √
y = 2 − x2 , − 2 < x < 2.
The solution of (13) is negative when x = 1 and is therefore of the form (16). Substi-
tuting x = 1 and y = −2 into (14) to satisfy the initial condition yields a2 = 5. Hence,
the solution of (13) is
√ √ √
y = − 5 − x2 , − 5 < x < 5.
··
^

(a)
1

x
−2 −1 1 2

−1

−2
(b)

√ √ √ √
Figure
√ 3. √(a) y = 2 − x2 , − 2 < x < 2; (b) y = − 5 − x2 ,
− 5<x< 5

Example 10. Solve x2 y 0 = 1 − x2 + y 2 − x2 y 2 , y(1) = 0.


Solution. First factor the right hand side to obtain
x2 y 0 = (1 − x2 )(1 + y 2 ).
9
Separate variables, integrate, and solve for y.
y0 1 − x2
= ,
1 + y2 x2
y0 1
2
= 2 − 1,
1+y x
−1
arctan(y) = − x + C,
x  
−1
y = tan −x+C .
x
Now solve for the initial condition, 0 = tan(−2 + C) to get C = 2 (or 2 + π, etc. . . ). The
solution we are seeking is, therefore,
 
−1
y = tan −x+2 .
x
··
^
Example 11. Find all solutions of
y 0 = 2xy 2 . (17)
Solution.
Here we must divide by p(y) = y 2 to separate variables. This isn’t legitimate if y is a
solution of (17) that equals zero for some value of x. One such solution can be found by
inspection: y ≡ 0. Now suppose y is a solution of (17) that isn’t identically zero. Since
y is continuous there must be an interval on which y is never zero. Since division by y 2
is legitimate for x in this interval, we can separate variables in (17) to obtain
y0
= 2x.
y2
Integrating this yields
1
− = x2 + c,
y
which is equivalent to
1
y=− . (18)
x2 +c
We’ve now shown that if y is a solution of (17) that is not identically zero, then y
must be of the form (18). By substituting (18) into (17), you can verify that (18) is a
solution of (17). Thus, solutions of (17) are y ≡ 0 and the functions of the form (18).
Note that the solution y ≡ 0 isn’t of the form (18) for any value of c.
Figure 7 shows a direction field and some integral curves for (17)
··
^
Example 12. Find all solutions of
1
y 0 = x(1 − y 2 ). (19)
2
Solution.
10
y
2

1.5

0.5

−0.5

−1

−1.5

−2 x
−2 −1.5 −1 −0.5 0 0.5 1 1.5 2

Figure 4. A direction field and integral curves for y 0 = 2xy 2

Here we must divide by p(y) = 1 − y 2 to separate variables. This isn’t legitimate if y


is a solution of (19) that equals ±1 for some value of x. Two such solutions can be found
by inspection: y ≡ 1 and y ≡ −1. Now suppose y is a solution of (19) such that 1 − y 2
isn’t identically zero. Since 1 − y 2 is continuous there must be an interval on which 1 − y 2
is never zero. Since division by 1 − y 2 is legitimate for x in this interval, we can separate
variables in (19) to obtain
2y 0
= −x.
y2 − 1
A partial fraction examplepansion on the left yields
 
1 1
− y 0 = −x,
y−1 y+1
and integrating yields
2

y − 1
ln = − x + k;
y + 1 2
hence,
y − 1 k −x2 /2
y + 1 = e e .

Since y(x) 6= ±1 for x on the interval under discussion, the quantity (y − 1)/(y + 1) can’t
change sign in this interval. Hence, we can rewrite the last equation as
y−1 2
= ce−x /2 ,
y+1
where c = ±ek , depending upon the sign of (y − 1)/(y + 1) on the interval. Solving for y
yields
2
1 + ce−x /2
y= . (20)
1 − ce−x2 /2
11
We’ve now shown that if y is a solution of (19) that is not identically equal to ±1,
then y must be as in (20). By substituting (20) into (19) you can verify that (20) is a
solution of (19). Thus, the solutions of (19) are y ≡ 1, y ≡ −1 and the functions of the
form (20). Note that the constant solution y ≡ 1 can be obtained from this formula by
taking c = 0; however, the other constant solution, y ≡ −1, can’t be obtained in this
way.
Figure 5 shows a direction field and some integrals for (19).

y
3

−1

−2

−3 x
−2 −1.5 −1 −0.5 0 0.5 1 1.5 2

x(1 − y 2 )
0
Figure 5. A direction field and integral curves for y =
2
··
^

Implicit Solutions of Separable Equations


In Examples 8 and 9 we were able to solve the equation H(y) = G(x) + c to obtain
explicit formulas for solutions of the given separable differential equations. As we’ll see
in the next examplea, this isn’t always possible. In this situation we must broaden our
definition of a solution of a separable equation. The next theorem provides the basis for
this modification. We omit the proof, which requires a result from advanced calculus
called as the implicit function theorem.
Theorem 13. Suppose g = g(x) is continous on (a, b) and h = h(y) are continuous
on (c, d). Let G be an antiderivative of g on (a, b) and let H be an antiderivative of h on
(c, d). Let x0 be an arbitrary point in (a, b), let y0 be a point in (c, d) such that h(y0 ) 6= 0,
and define and (x0 − ρ, x0 + ρ) = (−∞, ∞).)
c = H(y0 ) − G(x0 ). (21)
Then there’s a function y = y(x) defined on some open interval (a1 , b1 ), where a ≤ a1 <
x0 < b1 ≤ b, such that y(x0 ) = y0 and
H(y) = G(x) + c (22)
12
for a1 < x < b1 . Therefore y is a solution of the initial value problem
h(y)y 0 = g(x), y(x0 ) = x0 . (23)
It’s convenient to say that (22) with c arbitrary is an implicit solution of h(y)y 0 = g(x).
Curves defined by (22) are integral curves of h(y)y 0 = g(x). If c satisfies (21), we’ll say
that (22) is an implicit solution of the initial value problem (23). However, keep these
points in mind:
• For some choices of c there may not be any differentiable functions y that satisfy
(22).
• The function y in (22) (not (22) itself) is a solution of h(y)y 0 = g(x).
Example 14.

(1) Find implicit solutions of


2x + 1
y0 = . (24)
5y 4 + 1
(2) Find an implicit solution of
2x + 1
y0 = , y(2) = 1. (25)
5y 4 + 1
Solution. Separating variables yields
(5y 4 + 1)y 0 = 2x + 1.
Integrating yields the implicit solution
y 5 + y = x2 + x + c. (26)
of (24).
Imposing the initial condition y(2) = 1 in (26) yields 1 + 1 = 4 + 2 + c, so c = −4.
Thus,
y 5 + y = x2 + x − 4
is an implicit solution of the initial value problem (25). Although more than one differ-
entiable function y = y(x) satisfies 24) near x = 1, it can be shown that there’s only one
such function that satisfies the initial condition y(1) = 2.
Figure 6 shows a direction field and some integral curves for (24).
We’ve now shown that if y is a solution of (17) that is not identically zero, then y
must be of the form (18). By substituting (18) into (17), you can verify that (18) is a
solution of (17). Thus, solutions of (17) are y ≡ 0 and the functions of the form (18).
Note that the solution y ≡ 0 isn’t of the form (18) for any value of c.
··
Figure 7 shows a direction field and some integral curves for (17) ^

13
y
2

1.5

0.5

−0.5

−1 x
1 1.5 2 2.5 3 3.5 4

2x + 1
Figure 6. A direction field and integral curves for y 0 = t
5y 4 + 1

y
2

1.5

0.5

−0.5

−1

−1.5

−2 x
−2 −1.5 −1 −0.5 0 0.5 1 1.5 2

Figure 7. A direction field and integral curves for y 0 = 2xy 2

Exercises
x
(1) Solve y 0 = .
y
(2) Solve y 0 = x2 y.
dx
(3) Solve = (x2 − 1) t, for x(0) = 0.
dt
14
dx
(4) Solve = x sin(t), for x(0) = 1.
dt
dy
(5) Solve = xy + x + y + 1.
dx
(6) Solve xy 0 = y + 2x2 y, where y(1) = 1.
dy y2 + 1
(7) Solve = 2 , for y(0) = 1.
dx x +1
dy x2 + 1
(8) Find an implicit solution for = 2 , for y(0) = 1.
dx y +1
(9) Find an explicit solution for y 0 = xe−y , y(0) = 1.
(10) Find an explicit solution for xy 0 = e−y , for y(1) = 1.
2
(11) Find an explicit solution for y 0 = ye−x , y(0) = 1. It is alright to leave a definite
integral in your answer.
(12) Solve y 0 = 2xy.
(13) Solve x0 = 3xt2 − 3t2 , x(0) = 2.
1
(14) Find an implicit solution for x0 = 2 , x(0) = 1.
3x + 1
(15) Find an explicit solution to xy 0 = y 2 , y(1) = 1.
sin(x)
(16) Find an implicit solution to y 0 = .
cos(y)
In Exercises 17–21 find the general solution.
(17) y 0 + ay = 0 (a=constant)
(18) y 0 + 3x2 y = 0
(19) xy 0 + (ln x)y = 0
(20) xy 0 + 3y = 0
(21) x2 y 0 + y = 0

In Exercises
 22–27 solve the initial value problem.
1 + x
(22) y0 + y = 0, y(1) = 1
x 
0 1
(23) xy + 1 + y = 0, y(e) = 1
ln x
(24) xy 0 +(1 + x cot
 x)y = 0, y (π/2) = 2
2x
(25) y0 − y = 0, y(0) = 2
1 + x2
k
(26) y 0 + y = 0, y(1) = 3 (k= constant)
x
(27) y 0 + (tan kx)y = 0, y(0) = 2 (k= constant)

In Exercises 28 –31 find the general solution. Also, plot a direction field and
some integral curves on the rectangular region {−2 ≤ x ≤ 2, −2 ≤ y ≤ 2}.
(28) y 0 + 3y = 1
 
0 1 2
(29) y + −1 y =−
x x
0 −x2
(30) y + 2xy = xe
2x e−x
(31) y0 + y =
1 + x2 1 + x2

15
In Exercises 32 –40 find the general solution.
1 7
(32) y0 + y = 2 + 3
x x
4 1 sin x
(33) y0 + y= 5
+
x−1 (x − 1) (x − 1)4
2
(34) xy 0 + (1 + 2x2 )y = x3 e−x
2
(35) xy 0 + 2y = 2 + 1
x
(36) y 0 + (tan x)y = cos x
sin x
(37) (1 + x)y 0 + 2y =
1+x
(38) (x − 2)(x − 1)y 0 − (4x − 3)y = (x − 2)3
2
(39) y 0 + (2 sin x cos x)y = e− sin x
(40) x2 y 0 + 3xy = ex

In Exercises 41–45 solve the initial value problem and sketch the graph of the
solution.
(41) y 0 + 7y = e3x , y(0) = 0
2
(42) (1 + x2 )y 0 + 4xy = , y(0) = 1
1 + x2
2
(43) xy 0 + 3y = , y(−1) = 0
x(1 + x2 )
(44) y 0 + (cot x)y = cos x, y (π/2) = 1
1 2
(45) y 0 + y = 2 + 1, y(−1) = 0
x x
In Exercises 46–53 solve the initial value problem.
1 sin x
(46) (x − 1)y 0 + 3y = 3
+ , y(0) = 1
(x − 1) (x − 1)2
(47) xy 0 + 2y = 8x2 , y(1) = 3
(48) xy 0 − 2y = −x2 , y(1) = 1
(49) y 0 + 2xy = x, y(0) = 3
0 1 + (x − 1) sec2 x
(50) (x − 1)y + 3y = , y(0) = −1
(x − 1)3
1 + 2x2
(51) (x + 2)y 0 + 4y = , y(−1) = 2
x(x + 2)3
(52) (x2 − 1)y 0 − 2xy = x(x2 − 1), y(0) = 4
(53) (x2 − 5)y 0 − 2xy = −2x(x2 − 5), y(2) = 7

In Exercises 54–58 solve the initial value problem and leave the answer in a form
involving a definite integral.
(54) y 0 + 2xy = x2 , y(0) = 3
1 sin x
(55) y 0 + y = 2 , y(1) = 2
x x
0 e−x tan x
(56) y +y = , y(1) = 0
x
2x ex
(57) y0 + y = , y(0) = 1
1 + x2 (1 + x2 )2
2
(58) xy 0 + (x + 1)y = ex , y(1) = 2

16
(59) (a) Plot a direction field and some integral curves for
xy 0 − 2y = −1 (A)
on the rectangular region {−1 ≤ x ≤ 1, −.5 ≤ y ≤ 1.5}. What do all the
integral curves have in common?
(b) Show that the general solution of (A) on (−∞, 0) and (0, ∞) is
y = 1/2 + cx2 .
(c) Show that y is a solution of (A) on (−∞, ∞) if and only if
1/2 + c1 x2 , x ≥ 0,
(
y=
1/2 + c2 x2 , x < 0,
where c1 and c2 are arbitrary constants.
(d) Conclude from c) that all solutions of (A) on (−∞, ∞) are solutions of the
initial value problem
xy 0 − 2y = −1, y(0) = 1/2.
(e) Use b) to show that if x0 6= 0 and y0 is arbitrary, then the initial value
problem
xy 0 − 2y = −1, y(x0 ) = y0
has infinitely many solutions on (−∞, ∞).
(60) Suppose f is continuous on an open interval (a, b) and α is a constant.
(a) Derive a formula for the solution of the initial value problem
y 0 + αy = f (x), y(x0 ) = y0 , (A)
where x0 is in (a, b) and y0 is an arbitrary real number.
(b) Suppose (a, b) = (a, ∞), α > 0 and lim f (x) = L. Show that if y is the
x→∞
solution of (A), then lim y(x) = L/α.
x→∞
(61) Assume that all functions in this exercise are defined on a common interval (a, b).
(a) Prove: If y1 and y2 are solutions of
y 0 + p(x)y = f1 (x)
and
y 0 + p(x)y = f2 (x)
respectively, and c1 and c2 are constants, then y = c1 y1 + c2 y2 is a solution
of
y 0 + p(x)y = c1 f1 (x) + c2 f2 (x).
(This is the principle of superposition.)
(b) Use a) to show that if y1 and y2 are solutions of the nonhomogeneous equa-
tion
y 0 + p(x)y = f (x), (A)
then y1 − y2 is a solution of the homogeneous equation
y 0 + p(x)y = 0. (B)
(c) Use (a)to show that if y1 is a solution of (A) and y2 is a solution of (B),
then y1 + y2 is a solution of (A).
17
(62) Some nonlinear equations can be transformed into linear equations by changing
the dependent variable. Show that if
g 0 (y)y 0 + p(x)g(y) = f (x)
where y is a function of x and g is a function of y, then the new dependent
variable z = g(y) satisfies the linear equation
z 0 + p(x)z = f (x).
(63) Solve by the method discussed in Exercise 62.

a) (sec2 y)y 0 − 3 tan


 y = −1 xy 0
c) + 2 ln y = 4x2
2 2 1 y
b) ey 2yy 0 + = 2 y0 1
x x d) − = −3/x2
(1 + y) 2 x(1 + y)

(64) We’ve shown that if p and f are continuous on (a, b) then every solution of
y 0 + p(x)y = f (x) (A)
on (a, b) can be written as y = uy1 , where y1 is a nontrivial solution of the
complementary equation for (A) and u0 = f /y1 . Now suppose f , f 0 , . . . , f (m)
and p, p0 , . . . , p(m−1) are continuous on (a, b), where m is a positive integer, and
define
f0 = f,
0
fj = fj−1 + pfj−1 , 1 ≤ j ≤ m.
Show that
u(j+1) = fj /y1 , 0 ≤ j ≤ m.

Linear equations and the integrating factor


One of the most important types of equations we will learn how to solve are the so-
called linear equations. In fact, the majority of the course is about linear equations. In
this section we focus on the first order linear equation. A first order equation is linear if
we can put it into the form:
y 0 + p(x)y = f (x). (27)
0 0
The word linear means linear in y and y ; no higher powers nor functions of y or y appear.
The dependence on x can be more complicated.
Solutions of linear equations have nice properties. For instance, the solution exists
wherever p(x) and f (x) are defined, and has the same regularity (read: it is just as nice).
But most importantly for us right now, there is a method for solving linear first order
equations.
The trick is to rewrite the left hand side of (27) as a derivative of a product of y with
another function. To this end we find a function r(x) such that
dh i
r(x)y 0 + r(x)p(x)y = r(x)y .
dx
This is the left hand side of (27) multiplied by r(x). So if we multiply (27) by r(x), we
obtain
dh i
r(x)y = r(x)f (x).
dx
18
Now we integrate both sides. The right hand side does not depend on y and the left hand
side is written as a derivative of a function. Afterwards, we solve for y. The function r(x)
is called the integrating factor and the method is called the integrating factor method.
We are looking for a function r(x), such that if we differentiate it, we get the same
function back multiplied by p(x). That seems like a job for the exampleponential function!
Let R
r(x) = e p(x) dx .
We compute:
y 0 + p(x)y = f (x),
R R R
p(x) dx 0
e y + e p(x) dx p(x)y = e p(x) dx f (x),
d h R p(x) dx i R
e y = e p(x) dx f (x),
dx Z
R R
p(x) dx p(x) dx
e y= e f (x) dx + C,
R
 Z R 
− p(x) dx p(x) dx
y=e e f (x) dx + C .

Of course, to get a closed form formula for y, we need to be able to find a closed form
formula for the integrals appearing above.
Example 15. Solve
2
y 0 + 2xy = ex−x , y(0) = −1.
2
Solution. First note that p(x) = 2x and f (x) = ex−x . The integrating factor is
2
R
r(x) = e p(x) dx = ex . We multiply both sides of the equation by r(x) to get
2 2 2 2
ex y 0 + 2xex y = ex−x ex ,
d h x2 i
e y = ex .
dx
We integrate
2
ex y = ex + C,
2 2
y = ex−x + Ce−x .
Nexamplet, we solve for the initial condition −1 = y(0) = 1+C, so C = −2. The solution
is
2 2
y = ex−x − 2e−x .
··
^
Example 16. By using the variation of parameters find the general solution of
y 0 + 2y = x3 e−2x . (28)
By applying (a) of Example 3 with a = −2, we see that y1 = e−2x is a solution of the
complementary equation y 0 + 2y = 0. Therefore, we seek solutions of (28) in the form
y = ue−2x , so that
y 0 = u0 e−2x − 2ue−2x and y 0 + 2y = u0 e−2x − 2ue−2x + 2ue−2x = u0 e−2x . (29)
Hence, y is a solution of (28) if and only if
u0 e−2x = x3 e−2x or, equivalently, u0 = x3 .
19
y
2

1.5

0.5

−0.5

−1

−1.5

−2 x
−1 −0.8 −0.6 −0.4 −0.2 0 0.2 0.4 0.6 0.8 1

Figure 8. A direction field and integral curves for y 0 + 2y = x2 e−2x

Thus,
x4
u= + c,
4
and
y = ue−2x = e−2x x4 /4 + c


is the general solution of (28).


··
Figure 8 shows a direction field and some integral curves for (28). ^
Example 17.
(1) By using the variation of parameters, find the general solution
y 0 + (cot x)y = x csc x. (30)
(2) Solve the initial value problem
y 0 + (cot x)y = x csc x, y(π/2) = 1. (31)
Solution.
Here p(x) = cot x and f (x) = x csc x are both continuous except at the points x = rπ,
where r is an integer. Therefore we seek solutions of (30) on the intervals (rπ, (r + 1)π).
We need a nontrival solution y1 of the complementary equation; thus, y1 must satisfy
y10 + (cot x)y1 = 0, which we rewrite as
y10 cos x
= − cot x = − . (32)
y1 sin x
Integrating this yields
ln |y1 | = − ln | sin x|,
20
where we take the constant of integration to be zero since we need only one function that
satisfies (32). Clearly y1 = 1/ sin x is a suitable choice. Therefore we seek solutions of
(30) in the form
u
y= ,
sin x
so that
u0 u cos x
y0 = − (33)
sin x sin2 x
and
u0
y 0 + (cot x)y = − u cos x/ sin2 x + u cot x/ sin x
sin x
u0 u cos x u cos x
= − + (34)
sin x sin2 x sin2 x
u0
= .
sin x
Hence, y is a solution of (30) if and only if
u0 x
= x csc x = or, equivalently, u0 = x.
sin x sin x
Integrating this yields
u x2 c
u = x2 /2 + c, and y= = + . (35)
sin x 2 sin x sin x
is the general solution of (30) on every interval (rπ, (r + 1)π) (r =integer).
Imposing the initial condition y(π/2) = 1 in (35) yields
1 = π 2 /8 + c or c = 1 − π 2 /8.
Thus,
x2 (1 − π 2 /8)
y= +
2 sin x sin x
is a solution of (31). The interval of validity of this solution is (0, π); Figure 9 shows its
··
graph. ^

15

10

x
1 2 3

−5

− 10

− 15

Figure 9. Solution of y 0 + (cot x)y = x csc x, y(π/2) = 1

21
Remark 18. Let us summarize the method of variation of parameters for solving

y 0 + p(x)y = f (x) (36)


as follows:
(1) Find a function y1 such that
y10
= −p(x).
y1
For convenience, take the constant of integration to be zero.
(2) Write
y = uy1 (37)
to remind yourself of what you’re doing.
(3) Write u0 y1 = f and solve for u0 ; thus, u0 = f /y1 .
(4) Integrate u0 to obtain u, with an arbitrary constant of integration.
(5) Substitute u into (37) to obtain y.

To solve an equation written as


P0 (x)y 0 + P1 (x)y = F (x),
we recommend that you divide through by P0 (x) to obtain an equation of the form (36)
and then follow this procedure.

Solutions in Integral Form


The method of variation of parameters leads to this theorem.
Theorem 19. Suppose p and f are continuous on an open interval (a, b), and let y1
be any nontrivial solution of the complementary equation
y 0 + p(x)y = 0
on (a, b). Then:
(1) The general solution of the nonhomogeneous equation
y 0 + p(x)y = f (x) (38)
on (a, b) is  Z 
y = y1 (x) c + f (x)/y1 (x) dx . (39)

(2) If x0 is an arbitrary point in (a, b) and y0 is an arbitrary real number, then the
initial value problem
y 0 + p(x)y = f (x), y(x0 ) = y0
has the unique solution
 Z x 
y0 f (t)
y = y1 (x) + dt
y1 (x0 ) x0 y1 (t)
on (a, b).
Proof. To show that (39) is the general solution of (38) on (a, b), we must prove
that:
(i) If c is any constant, the function y in (39) is a solution of (38) on (a, b).
(i) If y is a solution of (38) on (a, b) then y is of the form (39) for some constant c.
22
To prove i), we first observe that any function of the form (39) is defined on (a, b),
since p and f are continuous on (a, b). Differentiating (39) yields
 Z 
0 0
y = y1 (x) c + f (x)/y1 (x) dx + f (x).

Since y10 = −p(x)y1 , this and (39) imply that


 Z 
0
y = −p(x)y1 (x) c + f (x)/y1 (x) dx + f (x)

= −p(x)y(x) + f (x),
which implies that y is a solution of (38).
To prove ii), suppose y is a solution of (38) on (a, b). From the proof of Theorem ??,
we know that y1 has no zeros on (a, b), so the function u = y/y1 is defined on (a, b).
Moreover, since
y 0 = −py + f and y10 = −py1 ,

u0 = y1 y 0 − y10 y/y12

= y1 (−py + f ) − (−py1 )y/y12 = f /y1 .


Integrating u0 = f /y1 yields
 Z 
u = c + f (x)/y1 (x) dx ,

which implies (39), since y = uyZ1 .


b) We’ve proved a), where f (x)/y1 (x) dx in (39) is an arbitrary antiderivative of
f /y1 . Now it’s convenient to choose the antiderivative that equals zero when x = x0 , and
write the general solution of (38) as
 Z x 
y = y1 (x) c + f (t)/y1 (t) dt .
x0

Since  Z x0 
y(x0 ) = y1 (x0 ) c + f (t)/y1 (t) dt = cy1 (x0 ),
x0
we see that y(x0 ) = y0 if and only if c = y0 /y1 (x0 ).
Sometimes the integrals that arise in solving a linear first order equation can’t be evalu-
ated in terms of elementary functions. In this case the solution must be left in terms of
an integral.
Example 20. (1) Find the general solution of
y 0 − 2xy = 1.
(2) Solve the initial value problem
y 0 − 2xy = 1, y(0) = y0 . (40)
Solution. To apply variation of parameters, we need a nontrivial solution y1 of the
complementary equation; thus, y10 − 2xy1 = 0, which we rewrite as
y10
= 2x.
y1
23
Integrating this and taking the constant of integration to be zero yields
2
ln |y1 | = x2 , so |y1 | = ex .
2 2
We choose y1 = ex and seek solutions of (40) in the form y = uex , where
2 2
u0 ex = 1, so u0 = e−x .
Therefore Z
2
u=c+ e−x dx,

but we can’t simplify the integral on the right because there’s no elementary function
2
with derivative equal to e−x . Therefore the best available form for the general solution
of (40) is
 Z 
x2 x2 −x2
y = ue = e c + e dx . (41)

Since the initial condition in (40) is imposed at x0 = 0, it is convenient to rewrite


(41) as
 Z x  Z 0
x2 −t2 2
y=e c+ e dt , since e−t dt = 0.
0 0
Setting x = 0 and y = y0 here shows that c = y0 . Therefore the solution of the initial
value problem is
 Z x 
x2 −t2
y=e y0 + e dt . (42)
0

For a given value of y0 and each fixed x, the integral on the right can be evaluated
by numerical methods. An alternate procedure is to apply the numerical integration
procedures discussed in Chapter 3 directly to the initial value problem (40). Figure 10
··
shows graphs of of (42) for several values of y0 . ^

Figure 10. Solutions of y 0 − 2xy = 1, y(0) = y0

24
Example 21. Solve the initial value problem
y 0 = 2xy 2 , y(0) = y0
and determine the interval of validity of the solution.
Solution.
First suppose y0 6= 0. From exampleample 11, we know that y must be of the form
y = −1/x2 + c. (43)
Imposing the initial condition shows that c = −1/y0 . Substituting this into (43) and
rearranging terms yields the solution
y = y0 /1 − y0 x2 .
This is also the solution if y0 = 0. If y0 < 0, the denominator isn’t zero for any value
of x, √ the solution is valid on (−∞, ∞). If y0 > 0, the solution is valid only on
so the √
(−1/ y0 , 1/ y0 ).
··
^
Exercises
(1) Solve
2
y 0 + 2xy = ex−x , y(0) = −1.
(2) Write the solution of the following problem as a definite integral, but try to
simplify as far as you can. You will not be able to find the solution in closed
form.
2
y 0 + y = ex −x , y(0) = 10.
0
(3) Solve y + xy = x.
(4) Solve y 0 + 6y = ex .
3
(5) Solve y 0 + 3x2 y = sin(x) e−x , with y(0) = 1.
(6) Solve y 0 + cos(x)y = cos(x).
1
(7) Solve 2 y 0 + xy = 3, with y(0) = 0.
x +1
(8) Solve y 0 + 3x2 y = x2 .
π
(9) Solve y 0 + 2 sin(2x)y = 2 sin(2x), y( ) = 3.
2
In Exercises 10–15 find all solutions.
3x2 + 2x + 1
(10) y0 =
y−2
(11) (sin x)(sin y) + (cos y)y 0 = 0
(12) xy 0 + y 2 + y = 0
(13) y 0 ln |y| + x2 y = 0
(2x + 1)y
(14) (3y 3 + 3y cos y + 1)y 0 + =0
1 + x2
(15) x2 yy 0 = (y 2 − 1)3/2

In Exercises 16–19 find all solutions. Also, plot a direction field and some integral
curves on the indicated rectangular region.
(16) y 0 = x2 (1 + y 2 ); {−1 ≤ x ≤ 1, −1 ≤ y ≤ 1}
(17) y 0 (1 + x2 ) + xy = 0; {−2 ≤ x ≤ 2, −1 ≤ y ≤ 1}
(18) y 0 = (x − 1)(y − 1)(y − 2); {−2 ≤ x ≤ 2, −3 ≤ y ≤ 3}
25
(19) (y − 1)2 y 0 = 2x + 3; {−2 ≤ x ≤ 2, −2 ≤ y ≤ 5}

In Exercises 20 and 21 solve the initial value problem.


x2 + 3x + 2
(20) y 0 = , y(1) = 4
y−2
(21) y 0 + x(y 2 + y) = 0, y(2) = 1

In Exercises 22–25 solve the initial value problem and graph the solution.
(22) (3y 2 + 4y)y 0 + 2x + cos x = 0, y(0) = 1
(y + 1)(y − 1)(y − 2)
(23) y 0 + = 0, y(1) = 0
x+1
(24) y 0 + 2x(y + 1) = 0, y(0) = 2
(25) y 0 = 2xy(1 + y 2 ), y(0) = 1

In Exercises 26–32 solve the initial value problem and find the interval of validity
of the solution.
(26) y 0 (x2 + 2) + 4x(y 2 + 2y + 1) = 0, y(1) = −1
(27) y 0 = −2x(y 2 − 3y + 2), y(0) = 3
2
(28) y 0 = , y(2) = 0
1 + 2y
(29) y 0 = 2y − y 2 , y(0) = 1
(30) x + yy 0 = 0, y(3) = −4
(31) y 0 + x2 (y + 1)(y − 2)2 = 0, y(4) = 2
(32) (x + 1)(x − 2)y 0 + y = 0, y(1) = −3
(1 + y 2 )
(33) Solve y 0 = explicitly.
√ (1 + x2 )p
(34) Solve y 0 1 − x2 + 1 − y 2 = 0 explicitly.
cos x
(35) Solve y 0 = , y(π) = π/2 explicitly.
sin y
(36) Solve the initial value problem
y 0 = ay − by 2 , y(0) = y0 .
Discuss the behavior of the solution if a) y0 ≥ 0; b) y0 < 0.
(37) The population P = P (t) of a species satisfies the logistic equation
P 0 = aP (1 − αP )
and P (0) = P0 > 0. Find P for t > 0, and find lim P (t).
t→∞
(38) An epidemic spreads through a population at a rate proportional to the product
of the number of people already infected and the number of people susceptible,
but not yet infected. Therefore, if S denotes the total population of susceptible
people and I = I(t) denotes the number of infected people at time t, then
I 0 = rI(S − I),
where r is a positive constant. Assuming that I(0) = I0 , find I(t) for t > 0, and
show that lim I(t) = S.
t→∞
(39) By plotting direction fields and solutions of
y 0 = qy − y 3 ,
26
convince yourself that q0 = 0 is a bifurcation value of q for this equation. exam-
pleplain what makes you draw this conclusion.
(40) Suppose a disease spreads according to the model of exampleercise 38, but there’s
a medication that cures the infected population at a constant rate of q individuals
per unit time, where q > 0. Then the equation for the number of infected
individuals becomes
I 0 = rI(S − I) − q.
Assuming that I(0) = I0 > 0, use the results of exampleercise ?? to describe
what happens as t → ∞.
(41) Assuming that p 6≡ 0, state conditions under which the linear equation
y 0 + p(x)y = f (x)
is separable. If the equation satisfies these conditions, solve it by separation of
variables and by the method developed in Section 2.1.

Solve the equations in exampleercises 42–45 using variation of parameters fol-


lowed by separation of variables.

2xe−x (x + 1)e4x
(42) y 0 + y = (44) y 0 − y =
1 + yex (y + ex )2
xe2x
x6 (45) y 0 − 2y =
(43) xy 0 − 2y = 1 − ye−2x
y + x2

(46) Use variation of parameters to show that the solutions of the following equations
are of the form y = uy1 , where u satisfies a separable equation u0 = g(x)p(u).
Find y1 and g for each equation.
a) xy 0 + y = h(x)p(xy)
b) xy 0 − y = h(x)p (y/x)
c) y 0 + y = h(x)p(ex y)
d) xy 0 + ry = h(x)p(xr y)
v 0 (x)
e) y 0 + y = h(x)p (v(x)y)
v(x)
Exact Differential Equations
In this section it’s convenient to write first order differential equations in the form
M (x, y) dx + N (x, y) dy = 0. (44)
This equation can be interpreted as
M (x, y) + N (x, y) dy/dx = 0, (45)
where x is the independent variable and y is the dependent variable, or as
M (x, y) dx/dy + N (x, y) = 0, (46)
where y is the independent variable and x is the dependent variable. Since the solutions
of (45) and (46) will often have to be left in implicit, form we’ll say that F (x, y) = c is an
implicit solution of (44) if every differentiable function y = y(x) that satisfies F (x, y) = c
is a solution of (45) and every differentiable function x = x(y) that satisfies F (x, y) = c
is a solution of (46).
Here are some examples:
27
Equation (44) Equation (45) Equation (46)

dy dx
3x2 y 2 dx + 2x3 y dy = 0 3x2 y 2 + 2x3 y =0 3x2 y 2 + 2x3 y = 0
dx dy
dy dx
(x2 + y 2 ) dx + 2xy dy = 0 (x2 + y 2 ) + 2xy =0 (x2 + y 2 ) + 2xy = 0
dx dy
dy dx
3y sin x dx − 2xy cos x dy = 0 3y sin x − 2xy cos x = 0 3y sin x − 2xy cos x = 0
dx dy
Note that a separable equation can be written as (44) as
M (x) dx + N (y) dy = 0.
We’ll develop a method for solving (44) under appropriate assumptions on M and
N . This method is an extension of the method of separation of variables (Exercise 44).
Before stating it we consider an example.
Example 22. Show that
x4 y 3 + x2 y 5 + 2xy = c (47)
is an implicit solution of
(4x3 y 3 + 2xy 5 + 2y) dx + (3x4 y 2 + 5x2 y 4 + 2x) dy = 0. (48)
Solution.
Regarding y as a function of x and differentiating (47) implicitly with respect to x
yields
dy
(4x3 y 3 + 2xy 5 + 2y) + (3x4 y 2 + 5x2 y 4 + 2x) = 0.
dx
Similarly, regarding x as a function of y and differentiating (47) implicitly with respect
to y yields
dx
(4x3 y 3 + 2xy 5 + 2y) + (3x4 y 2 + 5x2 y 4 + 2x) = 0.
dy
Thus, (47) is an implicit solution of (48) in either of its two possible interpretations.
··
^
You may think this example is pointless, since concocting a differential equation that
has a given implicit solution isn’t particularly interesting. However, it illustrates the next
important theorem, which we’ll prove by using implicit differentiation, as in Example 22.
Theorem 23. If F = F (x, y) has continuous partial derivatives Fx and Fy , then
F (x, y) = c (c=constant), (49)
is an implicit solution of the differential equation
Fx (x, y) dx + Fy (x, y) dy = 0. (50)
Proof. Regarding y as a function of x and differentiating (49) implicitly with respect
to x yields
Fx (x, y) + Fy (x, y) dy/dx = 0.
28
On the other hand, regarding x as a function of y and differentiating (49) implicitly with
respect to y yields
Fx (x, y) dx/dy + Fy (x, y) = 0.
Thus, (49) is an implicit solution of (50) in either of its two possible interpretations.

Definition. We’ll say that the equation


M (x, y) dx + N (x, y) dy = 0 (51)
is exact on an an open rectangle R if there’s a function F = F (x, y) such Fx and Fy are
continuous, and
Fx (x, y) = M (x, y) and Fy (x, y) = N (x, y) (52)
for all (x, y) in R. This usage of “exact” is related to its usage in calculus, where the
expression
Fx (x, y) dx + Fy (x, y) dy
(obtained by substituting (52) into the left side of (51)) is the exact differential of F .

Example 22 shows that it’s easy to solve (51) if it’s exact and we know a function F
that satisfies (52). The important questions are:
Question 1. Given an equation (51), how can we determine whether it’s exact?
Question 2. If (51) is exact, how do we find a function F satisfying (52)?
To discover the answer to Question 1, assume that there’s a function F that satisfies
(52) on some open rectangle R, and in addition that F has continuous mixed partial
derivatives Fxy and Fyx . Then a theorem from calculus implies that
Fxy = Fyx . (53)
If Fx = M and Fy = N , differentiating the first of these equations with respect to y and
the second with respect to x yields
Fxy = My and Fyx = Nx . (54)
From (53) and (54), we conclude that a necessary condition for exactness is that My = Nx .
This motivates the next theorem, which we state without proof.
Theorem 24. [The Exatness Condition] Suppose M and N are continuous and have
continuous partial derivatives My and Nx on an open rectangle R. Then
M (x, y) dx + N (x, y) dy = 0
is exact on R if and only if
My (x, y) = Nx (x, y) (55)
for all (x, y) in R..
To help you remember the exactness condition, observe that the coefficients of dx and
dy are differentiated in (55) with respect to the “opposite” variables; that is, the coefficient
of dx is differentiated with respect to y, while the coefficient of dy is differentiated with
respect to x.
Example 25. Show that the equation
3x2 y dx + 4x3 dy = 0
is not exact on any open rectangle.
29
Solution. Here,
M (x, y) = 3x2 y and N (x, y) = 4x3
so
My (x, y) = 3x2 and Nx (x, y) = 12x2 .
Hence, My = Nx on the line x = 0, but not on any open rectangle, so there’s no function F
such that Fx (x, y) = M (x, y) and Fy (x, y) = N (x, y) for all (x, y) on any open rectangle.
The next example illustrates two possible methods for finding a function F that
satisfies the condition Fx = M and Fy = N if M dx + N dy = 0 is exact.
··
^
Example 26. Solve
(4x3 y 3 + 3x2 ) dx + (3x4 y 2 + 6y 2 ) dy = 0. (56)
Solution. (Method 1) Here
M (x, y) = 4x3 y 3 + 3x2 , N (x, y) = 3x4 y 2 + 6y 2 ,
and
My (x, y) = Nx (x, y) = 12x3 y 2
for all (x, y). Hence, Theorem 24 implies that there’s a function F such that
Fx (x, y) = M (x, y) = 4x3 y 3 + 3x2 (57)
and
Fy (x, y) = N (x, y) = 3x4 y 2 + 6y 2 (58)
for all (x, y). To find F , we integrate (57) with respect to x to obtain
F (x, y) = x4 y 3 + x3 + ϕ(y), (59)
where ϕ(y) is the “constant” of integration. (Here ϕ is “constant” in that it’s independent
of x, the variable of integration.) If ϕ is any differentiable function of y then F satisfies
(57). To determine ϕ so that F also satisfies (58), assume that ϕ is differentiable and
differentiate F with respect to y. This yields
Fy (x, y) = 3x4 y 2 + ϕ0 (y).
Comparing this with (58) shows that
ϕ0 (y) = 6y 2 .
We integrate this with respect to y and take the constant of integration to be zero because
we’re interested only in finding some F that satisfies (57) and (58). This yields
ϕ(y) = 2y 3 .
Substituting this into (59) yields
F (x, y) = x4 y 3 + x3 + 2y 3 . (60)
Now Theorem 23 implies that
x4 y 3 + x3 + 2y 3 = c
is an implicit solution of (56). Solving this for y yields the explicit solution
1/3
y = c − x3 /2 + x4 .
··
^
30
y
1

0.8

0.6

0.4

0.2

−0.2

−0.4

−0.6

−0.8

−1 x
−1 −0.8 −0.6 −0.4 −0.2 0 0.2 0.4 0.6 0.8 1

Figure 11. A direction field and integral curves for (4x3 y 3 + 3x2 ) dx +
(3x4 y 2 + 6y 2 ) dy = 0

(Method 2) Instead of first integrating (57) with respect to x, we could begin by


integrating (58) with respect to y to obtain
F (x, y) = x4 y 3 + 2y 3 + ψ(x), (61)
where ψ is an arbitrary function of x. To determine ψ, we assume that ψ is differentiable
and differentiate F with respect to x, which yields
Fx (x, y) = 4x3 y 3 + ψ 0 (x).
Comparing this with (57) shows that
ψ 0 (x) = 3x2 .
Integrating this and again taking the constant of integration to be zero yields
ψ(x) = x3 .
Substituting this into (61) yields (60).
Figure 11 shows a direction field and some integral curves of (56),

Here’s a summary of the procedure used in Method 1 of Example 26.

Procedure For Solving An exampleact Equation


: Step 1. Check that the equation
M (x, y) dx + N (x, y) dy = 0 (62)
satisfies the exactness condition My = Nx . If not, don’t go further with this
procedure.
: Step 2. Integrate
∂F (x, y)/∂x = M (x, y)
31
with respect to x to obtain
F (x, y) = G(x, y) + ϕ(y), (63)
where G is an antiderivative of M with respect to x, and ϕ is an unknown
function of y.
: Step 3. Differentiate (63) with respect to y to obtain
∂F (x, y)/∂y = ∂G(x, y)/∂y + ϕ0 (y).
: Step 4. Equate the right side of this equation to N and solve for ϕ0 ; thus,
∂G(x, y)/∂y + ϕ0 (y) = N (x, y), so ϕ0 (y) = N (x, y) − ∂G(x, y)/∂y.
: Step 5. Integrate ϕ0 with respect to y, taking the constant of integration to be zero,
and substitute the result in (63) to obtain F (x, y).
: Step 6. Set F (x, y) = c to obtain an implicit solution of (62). If possible, solve for y
exampleplicitly as a function of x.
It’s a common mistake to omit Step 6. However, it’s important to include this step,
since F isn’t itself a solution of (62).
Many equations can be conveniently solved by either of the two methods used in
Example 26. However, sometimes the integration required in one approach is more dif-
ficult than in the other. In such cases we choose the approach that requires the easier
integration.
Example 27. Solve
dy −2x − y
= , y(0) = 1.
dx x−1
Solution. We write the equation as
dy
(2x + y) + (x − 1) = 0,
dx
so M = 2x + y and N = x − 1. Then
My = 1 = Nx .
The equation is exact. Integrating M in x, we find
F (x, y) = x2 + xy + A(y).
Differentiating in y and setting to N , we find
x − 1 = x + A0 (y).
So A0 (y) = −1, and A(y) = −y will work. Take F (x, y) = x2 + xy − y. We wish to
solve x2 + xy − y = C. First let us find C. As y(0) = 1 then F (0, 1) = C. Thus,
02 + 0 × 1 − 1 = C, so C = −1. Now we solve x2 + xy − y = −1 for y to get
−x2 − 1
y= .
x−1
··
^
Example 28. Solve
y x
− dx + 2 dy = 0, y(1) = 2.
x2
+y 2 x + y2
We leave to the reader to check that My = Nx .
32
This vector field (M, N ) is not conservative if considered as a vector field of the entire
plane minus the origin. The problem is that if the curve γ is a circle around the origin,
say starting at (1, 0) and ending at (1, 0) going counterclockwise, then if F exampleisted
we would examplepect
−y
Z Z
x
0 = F (1, 0) − F (1, 0) = Fx dx + Fy dy = 2 2
dx + 2 dy = 2π.
γ γ x +y x + y2
That is nonsense! We leave the computation of the path integral to the interested reader,
or you can consult your multivariable calculus texampletbook. So there is no potential
function F defined everywhere outside the origin (0, 0).
If we think back to the theorem, it does not guarantee such a function anyway. It
only guarantees a potential function locally, that is only in some region near the initial
point. As y(1) = 2 we start at the point (1, 2). Considering x > 0 and integrating M in
x or N in y, we find
y
F (x, y) = arctan .
x
y
The implicit solution is arctan = C. Solving, y = tan(C)x. That is, the solution is
x
a straight line. Solving y(1) = 2 gives us that tan(C) = 2, and so y = 2x is the desired
solution.
Example 29. Solve
dy
x2 + y 2 + 2y(x + 1) = 0.
dx
Check that it is exact. Let M = x2 + y 2 and N = 2y(x + 1). We follow the procedure
for exact equations
1
F (x, y) = x3 + xy 2 + A(y),
3
and
2y(x + 1) = 2xy + A0 (y).
1
Thus, A0 (y) = 2y or A(y) = y 2 and F (x, y) = x3 +xy 2 +y 2 . We try to solve F (x, y) = C.
3
We easily solve for y 2 and then just take the square root:
s
1
2 C − ( 3 )x 3
C − ( 31 )x3
y = , so y=± .
x+1 x+1
dy
When x = −1, the term in front of vanishes. You can also see that our solution is
dx
not valid in that case. However, one could in that case try to solve for x in terms of
1
y starting from the implicit solution x3 + xy 2 + y 2 = C. In this case the solution is
3
somewhat messy and we leave it as implicit.
··
^
Example 30. Solve the equation
(yexy tan x + exy sec2 x) dx + xexy tan x dy = 0. (64)
Solution. Note that My = Nx on any open rectangle where tan x and sec x are
defined. Here, we must find a function F such that
Fx (x, y) = yexy tan x + exy sec2 x (65)
33
and
Fy (x, y) = xexy tan x. (66)
It’s difficult to integrate (65) with respect to x, but easy to integrate (66) with respect
to y. This yields
F (x, y) = exy tan x + ψ(x). (67)
Differentiating this with respect to x yields
Fx (x, y) = yexy tan x + exy sec2 x + ψ 0 (x).
Comparing this with (65) shows that ψ 0 (x) = 0. Hence, ψ is a constant, which we can
take to be zero in (67), and
exy tan x = c
is an implicit solution of (64).
··
^
Remark 31. Attempting to apply our procedure to an equation that isn’t exact will
lead to failure in Step 4, since the function
N − ∂G/∂y
won’t be independent of x if My 6= Nx (Exercise 34), and thus can’t be the derivative of
a function of y alone. Here’s an example that illustrates this.
Example 32. Verify that the equation
3x2 y 2 dx + 6x3 y dy = 0 (68)
is not exact, and show that the procedure for solving exact equations fails when applied
to (68).
Solution. Here
My (x, y) = 6x2 y and Nx (x, y) = 18x2 y,
so (68) isn’t exact. Nevertheless, let’s try to find a function F such that
Fx (x, y) = 3x2 y 2 (69)
and
Fy (x, y) = 6x3 y. (70)
Integrating (69) with respect to x yields
F (x, y) = x3 y 2 + ϕ(y),
and differentiating this with respect to y yields
Fy (x, y) = 2x3 y + ϕ0 (y).
For this equation to be consistent with (70),
6x3 y = 2x3 y + ϕ0 (y),
or
ϕ0 (y) = 4x3 y.
This is a contradiction, since ϕ0 must be independent of x. Therefore the procedure fails.
··
^

Exercises
In Exerrcises 1–17 determine which equations are exact and solve them.
34
(1) 6x2 y 2 dx + 4x3 y dy = 0
(2) (3y cos x + 4xex + 2x2 ex ) dx + (3 sin x + 3) dy = 0
(3) 14x2 y 3 dx + 21x2 y 2 dy = 0
(4) (2x − 2y 2 ) dx + (12y 2 − 4xy) dy = 0
(5) (x + y)2 dx + (x + y)2 dy = 0
(6) (4x + 7y) dx + (3x + 4y) dy = 0
(7) (−2y 2 sin x + 3y 3 − 2x) dx + (4y cos x + 9xy 2 ) dy = 0
(8) (2x + y) dx + (2y + 2x) dy = 0
(9) (3x2 + 2xy + 4y 2 ) dx + (x2 + 8xy + 18y) dy = 0
(10) (2x2 + 8xy

2
 + y ) dx + (2x2
+ xy 3 /3) dy = 0
1 1
(11) + 2x dx + + 2y dy = 0
x y
(12) (y sin xy + xy 2 cos xy) dx + (x sin xy + xy 2 cos xy) dy = 0
x dx y dy
(13) 2 2 3/2
+ 2 =0
(x + y ) (x + y2 )3/2
x 2 2 2 2 x
(14) e (x y + 2xy ) + 6x dx + (2x ye + 2) dy = 0
2 +y 2
(15) x2 ex (2x2 + 3) + 4x dx + (x3 ex +y − 12y 2 ) dy = 0
(16) exy (x4 y + 4x3 ) + 3y dx + (x5 exy + 3x) dy = 0


(17) (3x2 cos xy − x3 y sin xy + 4x) dx + (8y − x4 sin xy) dy = 0

In Exercises 18–22 solve the initial value problem.


(18) (4x3 y 2 − 6x2 y − 2x − 3) dx + (2x4 y − 2x3 ) dy = 0, y(1) = 3
(19) (−4y cos x + 4 sin x cos x + sec2 x) dx + (4y − 4 sin x) dy = 0, y(π/4) = 0
(20) (y 3 − 1)ex dx + 3y 2 (ex + 1) dy = 0, y(0) = 0
(21) (sin x − y sin x − 2 cos x) dx + cos x dy = 0, y(0) = 1
(22) (2x − 1)(y − 1) dx + (x + 2)(x − 3) dy = 0, y(1) = −1
(23) Solve
dy −2x − y
= , y(0) = 1.
dx x−1
(24) Solve
y x
− 2 dx + dy = 0, y(1) = 2.
x + y2 x2 + y 2
(25) Solve
dy
x2 + y 2 + 2y(x + 1) = 0.
dx
(26) Solve the exact equation
(7x + 4y) dx + (4x + 3y) dy = 0.
Plot a direction field and some integral curves for this equation on the rectangle
{−1 ≤ x ≤ 1, −1 ≤ y ≤ 1}.
(27) Solve the exact equation
ex (x4 y 2 + 4x3 y 2 + 1) dx + (2x4 yex + 2y) dy = 0.
Plot a direction field and some integral curves for this equation on the rectangle
{−2 ≤ x ≤ 2, −1 ≤ y ≤ 1}.
35
(28) Plot a direction field and some integral curves for the exact equation
(x3 y 4 + x) dx + (x4 y 3 + y) dy = 0
on the rectangle {−1 ≤ x ≤ 1, −1 ≤ y ≤ 1}. (See Exercise 40a).
(29) Plot a direction field and some integral curves for the exact equation
(3x2 + 2y) dx + (2y + 2x) dy = 0
on the rectangle {−2 ≤ x ≤ 2, −2 ≤ y ≤ 2}. (See Exercise 40(b)).
(30) (a) Solve the exact equation
(x3 y 4 + 2x) dx + (x4 y 3 + 3y) dy = 0 (A)
implicitly.
(b) For what choices of (x0 , y0 ) does Theorem 40 imply that the initial value
problem
(x3 y 4 + 2x) dx + (x4 y 3 + 3y) dy = 0, y(x0 ) = y0 , (B)
has a unique solution on an open interval (a, b) that contains x0 ?
(c) Plot a direction field and some integral curves for (A) on a rectangular
region centered at the origin. What is the interval of validity of the solution
of (B)?
(31) (a) Solve the exact equation
(x2 + y 2 ) dx + 2xy dy = 0 (A)
implicitly.
(b) For what choices of (x0 , y0 ) does Theorem 40 imply that the initial value
problem
(x2 + y 2 ) dx + 2xy dy = 0, y(x0 ) = y0 , (B)
has a unique solution y = y(x) on some open interval (a, b) that contains
x0 ?
(c) Plot a direction field and some integral curves for (A). From the plot de-
termine, the interval (a, b) of b), the monotonicity properties (if any) of the
solution of (B), and lim y(x) and lim y(x).
x→a+ x→b−
(32) Find all functions M such that the equation is exact.
(a) M (x, y) dx + (x2 − y 2 ) dy = 0
(b) M (x, y) dx + 2xy sin x cos y dy = 0
(c) M (x, y) dx + (ex − ey sin x) dy = 0
(33) Find all functions N such that the equation is exact.
(a) (x3 y 2 + 2xy + 3y 2 ) dx + N (x, y) dy = 0
(b) (ln xy + 2y sin x) dx + N (x, y) dy = 0
(c) (x sin x + y sin y) dx + N (x, y) dy = 0
(34) Suppose M, N, and their partial derivatives are continuous on an open rectangle
R, and G is an antiderivative of M with respect to x; that is,
∂G/∂x = M.
Show that if My 6= Nx in R then the function
N − ∂G/∂y
is not independent of x.
36
(35) Prove: If the equations M1 dx + N1 dy = 0 and M2 dx + N2 dy = 0 are exact on
an open rectangle R, so is the equation
(M1 + M2 ) dx + (N1 + N2 ) dy = 0.
(36) Find conditions on the constants A, B, C, and D such that the equation
(Ax + By) dx + (Cx + Dy) dy = 0
is exact.
(37) Find conditions on the constants A, B, C, D, E, and F such that the equation
(Ax2 + Bxy + Cy 2 ) dx + (Dx2 + Exy + F y 2 ) dy = 0
is exact.
(38) Suppose M and N are continuous and have continuous partial derivatives My
and Nx that satisfy the exactness condition My = Nx on an open rectangle R.
Show that if (x, y) is in R and
Z x Z y
F (x, y) = M (s, y0 ) ds + N (x, t) dt,
x0 y0

then Fx = M and Fy = N .
(39) Under the assumptions of Exercise 38, show that
Z y Z x
F (x, y) = N (x0 , s) ds + M (t, y) dt.
y0 x0

(40) Use the method suggested by Exercise 38, with (x0 , y0 ) = (0, 0), to solve the
these exact equations:
(a) (x3 y 4 + x) dx + (x4 y 3 + y) dy = 0
(b) (x2 + y 2 ) dx + 2xy dy = 0
(c) (3x2 + 2y) dx + (2y + 2x) dy = 0
(41) Solve the initial value problem
2 2xy
y0 + y = − 2 , y(1) = −2.
x x + 2x2 y + 1
(42) Solve the initial value problem
3 2x4 (4x3 − 3y)
y0 − y = 5 , y(1) = 1.
x 3x + 3x3 + 2y
(43) Solve the initial value problem
!
x2
2 3x + 2ye
y 0 + 2xy = −e−x , y(0) = −1.
2x + 3yex2
(44) Rewrite the separable equation
h(y)y 0 = g(x) (A)
as an exact equation
M (x, y) dx + N (x, y) dy = 0. (B)
Show that applying the method of this section to (B) yields the same solutions
that would be obtained by applying the method of separation of variables to (A)
(45) Suppose all second partial derivatives of M = M (x, y) and N = N (x, y) are
continuous and M dx + N dy = 0 and −N dx + M dy = 0 are exact on an open
rectangle R. Show that Mxx + Myy = Nxx + Nyy = 0 on R.
37
(46) Suppose all second partial derivatives of F = F (x, y) are continuous and Fxx +
Fyy = 0 on an open rectangle R. (A function with these properties is said to be
harmonic; see also Exercise 45.) Show that −Fy dx + Fx dy = 0 is exact on R,
and therefore there’s a function G such that Gx = −Fy and Gy = Fx in R. (A
function G with this property is said to be a harmonic conjugate of F .)
(47) Verify that the following functions are harmonic, and find all their harmonic
conjugates. (See Exercise 46.)
(a) x2 − y 2 (b) ex cos y (c) x3 − 3xy 2
(d) cos x cosh y (e) sin x cosh y
Integrating Factors
We saw that if M , N , My and Nx are continuous and My = Nx on an open rectangle
R then
M (x, y) dx + N (x, y) dy = 0 (71)
is exact on R. Sometimes an equation that isn’t exampleact can be made exact by
multiplying it by an appropriate function. For instance,
(3x + 2y 2 ) dx + 2xy dy = 0 (72)
is not exact, since My (x, y) = 4y 6= Nx (x, y) = 2y in (72). However, multiplying (72) by
x yields
(3x2 + 2xy 2 ) dx + 2x2 y dy = 0, (73)
which is exact, since My (x, y) = Nx (x, y) = 4xy in (73). Solving (73) by the procedure
given in previous section yields the implicit solution
x3 + x2 y 2 = c.
Definition. A function µ = µ(x, y) is an integrating factor for (71) if
µ(x, y)M (x, y) dx + µ(x, y)N (x, y) dy = 0 (74)
is exact.
If we know an integrating factor µ for (71), we can solve the exact equation (74)
by the method of previous section. It would be nice if we could say that (71) and (74)
always have the same solutions, but this isn’t so. For example, a solution y = y(x) of
(74) such that µ(x, y(x)) = 0 on some interval a < x < b could fail to be a solution of
(71) (Exercise 10), while (71) may have a solution y = y(x) such that µ(x, y(x)) isn’t
even defined (Exercise 11). Similar comments apply if y is the independent variable and
x is the dependent variable in (71) and (74). However, if µ(x, y) is defined and nonzero
for all (x, y), (71) and (74) are equivalent; that is, they have the same solutions.

Finding Integrating Factors


By applying Theorem 24 (with M and N replaced by µM and µN ), we see that (74) is
exact on an open rectangle R if µM , µN , (µM )y , and (µN )x are continuous and
∂ ∂
(µM ) = (µN ) or, equivalently, µy M + µMy = µx N + µNx
∂y ∂x
on R. It’s better to rewrite the last equation as
µ(My − Nx ) = µx N − µy M, (75)
which reduces to the known result for exact equations; that is, if My = Nx then (75)
holds with µ = 1, so (71) is exact.
38
If µ(x, y) = P (x)Q(y), then µx (x, y) = P 0 (x)Q(y) and µy (x, y) = P (x)Q0 (y), so (75)
becomes
P (x)Q(y)(My − Nx ) = P 0 (x)Q(y)N − P (x)Q0 (y)M, (76)
or, after dividing through by P (x)Q(y),
P 0 (x) Q0 (y)
My − Nx = N− M. (77)
P (x) Q(y)
Now let
p(x) = P 0 (x)/P (x) and q(y) = Q0 (y)/Q(y),
so (77) becomes
My − Nx = p(x)N − q(y)M. (78)
We obtained (78) by assuming that M dx + N dy = 0 has an integrating factor
µ(x, y) = P (x)Q(y). However, we can now view (77) differently: If there are functions
p = p(x) and q = q(y) that satisfy (78) and we define
R R
p(x) dx q(y) dy
P (x) = ±e and Q(y) = ±e , (79)
then reversing the steps that led from (76) to (78) shows that µ(x, y) = P (x)Q(y) is an
integrating factor for M dx + N dy = 0. In using this result, we take the constants of
integration in (79) to be zero and choose the signs conveniently so the integrating factor
has the simplest form.
There’s no simple general method for ascertaining whether functions p = p(x) and
q = q(y) satisfying (78) exampleist. However, the next theorem gives simple sufficient
conditions for the given equation to have an integrating factor that depends on only one
of the independent variables x and y, and for finding an integrating factor in this case.
Theorem 33. Let M, N, My , and Nx be continuous on an open rectangle R. Then:
My − Nx
(1) If is independent of y on R and we define
N
My − Nx
p(x) =
N
then
R
p(x) dx
µ(x) = ±e (80)
is an integrating factor for
M (x, y) dx + N (x, y) dy = 0 (81)
on R.
Nx − My
(2) If is independent of x on R and we define
M
Nx − My
q(y) = ,
M
then
R
q(y) dy
µ(y) = ±e (82)
is an integrating factor for (81) on R.

39
Proof. (a) If (My −Nx )/N is independent of y, then (78) holds with p = (My −Nx )/N
and q ≡ 0. Thus,
R R
p(x) dx q(y) dy
P (x) = ±e and Q(y) = ±e = ±e0 = ±1,
so (80) is an integrating factor for (81) on R.
(b) If (Nx − My )/M is independent of x then eqrefeq:2.6.8 holds with p ≡ 0 and
q = (Nx − My )/M , and a similar argument shows that (82) is an integrating factor for
(81) on R.
The next two examples show how to apply Theorem 33.
Example 34. Find an integrating factor for the equation
(2xy 3 − 2x3 y 3 − 4xy 2 + 2x) dx + (3x2 y 2 + 4y) dy = 0 (83)
and solve the equation.
Solution. In (83)
M = 2xy 3 − 2x3 y 3 − 4xy 2 + 2x, N = 3x2 y 2 + 4y,
and
My − Nx = (6xy 2 − 6x3 y 2 − 8xy) − 6xy 2 = −6x3 y 2 − 8xy,
so (83) isn’t exact. However,
My − Nx 6x3 y 2 + 8xy
=− 2 2 = −2x
N 3x y + 4y
is independent of y, so Theorem 33a) applies with p(x) = −2x. Since
Z Z
p(x) dx = − 2x dx = −x2 ,
2
µ(x) = e−x is an integrating factor. Multiplying (83) by µ yields the exact equation
2 2
e−x (2xy 3 − 2x3 y 3 − 4xy 2 + 2x) dx + e−x (3x2 y 2 + 4y) dy = 0. (84)
To solve this equation, we must find a function F such that
2
Fx (x, y) = e−x (2xy 3 − 2x3 y 3 − 4xy 2 + 2x) (85)
and
2
Fy (x, y) = e−x (3x2 y 2 + 4y). (86)
Integrating (86) with respect to y yields
2
F (x, y) = e−x (x2 y 3 + 2y 2 ) + ψ(x). (87)
Differentiating this with respect to x yields
2
Fx (x, y) = e−x (2xy 3 − 2x3 y 3 − 4xy 2 ) + ψ 0 (x).
2 2
Comparing this with (85) shows that ψ 0 (x) = 2xe−x ; therefore, we can let ψ(x) = −e−x
in (87) and conclude that
2
e−x y 2 (x2 y + 2) − 1 = c


is an implicit solution of (84). It is also an implicit solution of (83).


Figure 12 shows a direction field and some integal curves for (83)
··
^
40
y
4

0 x

−1

−2

−3

−4
−2 −1.5 −1 −0.5 0 0.5 1 1.5 2

Figure 12. A direction field and integral curves for (2xy 3 −2x3 y 3 −4xy 2 +
2x) dx + (3x2 y 2 + 4y) dy = 0

Example 35. Find an integrating factor for


2xy 3 dx + (3x2 y 2 + x2 y 3 + 1) dy = 0 (88)
and solve the equation.
Solution. In (88),
M = 2xy 3 , N = 3x2 y 2 + x2 y 3 + 1,
and
My − Nx = 6xy 2 − (6xy 2 + 2xy 3 ) = −2xy 3 ,
so (88) isn’t exact. Moreover,
My − Nx 2xy 3
=− 2 2
N 3x y + x2 y 2 + 1
is not independent of y, so Theorem 33(a) does not apply. However, Theorem 33(b) does
apply, since
Nx − My 2xy 3
= =1
M 2xy 3
is independent of x, so we can take q(y) = 1. Since
Z Z
q(y) dy = dy = y,

µ(y) = ey is an integrating factor. Multiplying (88) by µ yields the exampleact equation


2xy 3 ey dx + (3x2 y 2 + x2 y 3 + 1)ey dy = 0. (89)
To solve this equation, we must find a function F such that
Fx (x, y) = 2xy 3 ey (90)
41
and
Fy (x, y) = (3x2 y 2 + x2 y 3 + 1)ey . (91)
Integrating (90) with respect to x yields
F (x, y) = x2 y 3 ey + ϕ(y). (92)
Differentiating this with respect to y yields
Fy = (3x2 y 2 + x2 y 3 )ey + ϕ0 (y),
and comparing this with (91) shows that ϕ0 (y) = ey . Therefore we set ϕ(y) = ey in (92)
and conclude that
(x2 y 3 + 1)ey = c
is an implicit solution of (89). It is also an implicit solution of (88). Figure 13 shows a
direction field and some integral curves for (88).

0 x

−1

−2

−3

−4
−4 −3 −2 −1 0 1 2 3 4

Figure 13. A direction field and integral curves for 2xy 3 ey dx + (3x2 y 2 +
x2 y 3 + 1)ey dy = 0

Theorem 33 does not apply in the next example but the more general argument that
led to Theorem 33 provides an integrating factor.
Example 36. Find an integrating factor for
(3xy + 6y 2 ) dx + (2x2 + 9xy) dy = 0 (93)
and solve the equation.
Solution. In (93)
M = 3xy + 6y 2 , N = 2x2 + 9xy,
and
My − Nx = (3x + 12y) − (4x + 9y) = −x + 3y.
Hence,
My − Nx −x + 3y Nx − My x − 3y
= and = 2 ,
M 3xy + 6y 2 N 2x + 9xy
42
so Theorem 33 does not apply.

Following the more general argument that led to Theorem 33, we look for functions
p = p(x) and q = q(y) such that
My − Nx = p(x)N − q(y)M ;
that is,
−x + 3y = p(x)(2x2 + 9xy) − q(y)(3xy + 6y 2 ).
Since the left side contains only first degree terms in x and y, we rewrite this equation as
xp(x)(2x + 9y) − yq(y)(3x + 6y) = −x + 3y.
This will be an identity if
xp(x) = A and yq(y) = B, (94)
where A and B are constants such that
−x + 3y = A(2x + 9y) − B(3x + 6y),
or, equivalently,
−x + 3y = (2A − 3B)x + (9A − 6B)y.
Equating the coefficients of x and y on both sides shows that the last equation holds for
all (x, y) if
2A − 3B = −1
9A − 6B = 3,
which has the solution A = 1, B = 1. Therefore (94) implies that
p(x) = 1/x and q(y) = 1/y.
Since Z Z
p(x) dx = ln |x| and q(y) dy = ln |y|,
we can let P (x) = x and Q(y) = y; hence, µ(x, y) = xy is an integrating factor. Multi-
plying (93) by µ yields the exact equation
(3x2 y 2 + 6xy 3 ) dx + (2x3 y + 9x2 y 2 ) dy = 0.
This equation has the implicit solution
x3 y 2 + 3x2 y 3 = c. (95)
This is also an implicit solution of (93). Since x ≡ 0 and y ≡ 0 satisfy (95), you should
check to see that x ≡ 0 and y ≡ 0 are also solutions of (93). (Why is it necesary to check
this?)
Figure 14 shows a direction field and integral curves for (93).
··
^
See Exercise 37 for a general discussion of equations like (93).
Example 37. The separable equation
− y dx + (x + x6 ) dy = 0 (96)
can be converted to the exact equation
dx dy
− 6
+ =0 (97)
x+x y
43
y

1.5

0.5

0 x

−0.5

−1

−1.5

−2
−4 −3 −2 −1 0 1 2 3 4

Figure 14. A direction field and integral curves for (3xy + 6y 2 ) dx +


(2x2 + 9xy) dy = 0

by multiplying through by the integrating factor


1
µ(x, y) = .
y(x + x6 )
However, to solve (97) by the method of previous section we would have to evaluate the
integral Z
dx
.
x + x6
Instead, we solve (96) explicitly for y by finding an integrating factor of the form µ(x, y) =
xa y b .

Solution. In (96)
M = −y, N = x + x6 ,
and
My − Nx = −1 − (1 + 6x5 ) = −2 − 6x5 .
We look for functions p = p(x) and q = q(y) such that
My − Nx = p(x)N − q(y)M ;
that is,
− 2 − 6x5 = p(x)(x + x6 ) + q(y)y. (98)
The right side will contain the term −6x5 if p(x) = −6/x. Then (98) becomes
−2 − 6x5 = −6 − 6x5 + q(y)y,
so q(y) = 4/y. Since
Z Z
6 1
p(x) dx = − dx = −6 ln |x| = ln 6 ,
x x
44
y

0.8

0.6

0.4

0.2

0 x

−0.2

−0.4

−0.6

−0.8

−1
−1 −0.8 −0.6 −0.4 −0.2 0 0.2 0.4 0.6 0.8 1

Figure 15. A direction field and integral curves for −y dx + (x + x6 ) dy = 0

and Z Z
q(y) dy = 4/y dy = 4 ln |y| = ln y 4 ,

we can take P (x) = x−6 and Q(y) = y 4 , which yields the integrating factor µ(x, y) =
x−6 y 4 . Multiplying (96) by µ yields the exampleact equation
y5
 4 
y 4
− 6 dx + +y dy = 0.
x x5
We leave it to you to use the method of the Section 2.5 to show that this equation has
the implicit solution
(y/x)5 + y 5 = k.
Solving for y yields
y = k 1/5 x(1 + x5 )−1/5 ,
which we rewrite as
y = cx(1 + x5 )−1/5
by renaming the arbitrary constant. This is also a solution of (96).
Figure 15 shows a direction field and some integral curves for (96).
··
^
Example 38. Solve
x2 + y 2 dy
+ 2y = 0.
x+1 dx
x2 + y 2
Solution. Let M = and N = 2y. Compute
x+1
2y 2y
My − Nx = −0= .
x+1 x+1
45
As this is not zero, the equation is not exact. We notice
My − Nx 2y 1 1
P (x) = = =
N x + 1 2y x+1
is a function of x alone. We compute the integrating factor
R
P (x) dx
e = eln(x+1) = x + 1.
We multiply our given equation by (x + 1) to obtain
dy
x2 + y 2 + 2y(x + 1) = 0,
dx
which is an exact equation that we solved in 29. The solution was
s
C − ( 13 )x3
y=± .
x+1
Example 39. Solve
dy
y 2 + (xy + 1) = 0.
dx
Solution. First compute
My − Nx = 2y − y = y.
As this is not zero, the equation is not exact. We observe
My − Nx y 1
Q(y) = = 2 =
M y y
is a function of y alone. We compute the integrating factor
R 1
e− Q(y) dy
= e− ln y = .
y
Therefore we look at the exact equation
xy + 1 dy
y+ = 0.
y dx
Check that this equation is exact. We follow the procedure for exampleact equations
F (x, y) = xy + A(y),
and
xy + 1 1
= x + = x + A0 (y). (99)
y y
1
Consequently A0 (y) = or A(y) = ln y. Thus F (x, y) = xy + ln y. It is not possible to
y
solve F (x, y) = C for y in terms of elementary functions, so let us be content with the
implicit solution:
xy + ln y = C.
··
^
Exercises
46
(1) Solve the following exact equations, implicit general solutions will suffice:
a) (2xy + x2 ) dx + (x2 + y 2 + 1) dy = 0
dy
b) x5 + y 5 =0
dx
dy
c) ex + y 3 + 3xy 2 =0
dx
d) (x + y) cos(x) + sin(x) + sin(x)y 0 = 0
(2) Find the integrating factor for the following equations making them into exact
equations:
y
a) exy dx + exy dy = 0
x
ex + y 3
b) dx + 3x dy = 0
y2
2x + 2y 2
c) 4(y 2 + x) dx + dy = 0
y
d) 2 sin(y) dx + x cos(y) dy = 0
dy
(3) Suppose you have an equation of the form: f (x) + g(y) = 0.
dx
a) Show it is exact.
b) Find the form of the potential function in terms of f and g.
(4) Suppose that we have the equation f (x) dx − dy = 0.
a) Is this equation exact?
b) Find the general solution using a definite integral.
(5) Find the potential function F (x, y) of the exact equation
1 + xy 1 
dx + + x dy = 0
x y
in two different ways.
a) Integrate M in terms of x and then differentiate in y and set to N .
b) Integrate N in terms of y and then differentiate in x and set to M .
(6) A function u(x, y) is said to be harmonic function if uxx + uyy = 0.
a) Show that −uy dx + ux dy = 0 is an exact equation. Therefore there exists
(at least locally) the so-called harmonic conjugate function v(x, y) such that
vx = −uy and vy = ux .
Verify that the following u are harmonic and find the corresponding harmonic
conjugates v:
b) u = 2xy
c) u = ex cos y
d) u = x3 − 3xy 2
(7) Solve the following exact equations, implicit general solutions will suffice:
a) cos(x) + yexy + xexy y 0 = 0
b) (2x + y) dx + (x − 4y) dy = 0
dy
c) ex + ey =0
dx
d) (3x + 3y) dx + (3y 2 + 3x) dy = 0
2

(8) Find the integrating factor for the following equations making them into exact
equations:
1
a) dx + 3y dy = 0
y
b) dx − e−x−y dy = 0
47
cos(x) 1  x
c) 2
+ dx + 2 dy = 0
y y y
y2 
d) 2y + dx + (2y + x) dy = 0
x
(9) a) Show that every separable equation y 0 = f (x)g(y) can be written as an exact
equation, and verify that it is indeed exact. b) Using this rewrite y 0 = xy as an
exact equation.
(10) (a) Verify that µ(x, y) = y is an integrating factor for
 
1
y dx + 2x + dy = 0 (A)
y
on any open rectangle that does not intersect the x axis or, equivalently,
that
y 2 dx + (2xy + 1) dy = 0 (B)
is exact on any such rectangle.
(b) Verify that y ≡ 0 is a solution of (B), but not of (A).
(c) Show that
y(xy + 1) = c (C)
is an implicit solution of (B), and explain why every differentiable function
y = y(x) other than y ≡ 0 that satisfies (C) is also a solution of (A).
(11) (a) Verify that µ(x, y) = 1/(x − y)2 is an integrating factor for
−y 2 dx + x2 dy = 0 (A)
on any open rectangle that does not intersect the line y = x or, equivalently,
that
−y 2 /(x − y)2 dx + x2 /(x − y)2 dy = 0 (B)
is exact on any such rectangle.
(b) Use Theorem 13 to show that
xy
=c (C)
(x − y)
is an implicit solution of (B), and explain why it’s also an implicit solution
of (A)
(c) Verify that y = x is a solution of (A), even though it can’t be obtained from
(C).
In Exercises 12–25 find an integrating factor; that is a function of only one
variable, and solve the given equation.
(12) y dx − x dy = 0
(13) 3x2 y dx + 2x3 dy = 0
(14) 2y 3 dx + 3y 2 dy = 0
(15) (5xy + 2y + 5) dx + 2x dy = 0
(16) (xy + x + 2y + 1) dx + (x + 1) dy = 0
(17) (27xy 2 + 8y 3 ) dx + (18x2 y + 12xy 2 ) dy = 0
(18) (6xy 2 + 2y) dx + (12x2 y + 6x + 3) dy = 0
1
(19) y 2 dx + xy 2 + 3xy + dy = 0
y
(20) (12x3 y + 24x2 y 2 ) dx + (9x4 + 32x3 y + 4y) dy = 0
(21) (x2 y + 4xy + 2y) dx + (x2 + x) dy = 0
(22) −y dx + (x4 − x) dy = 0
48
(23) cos x cos y dx + (sin x cos y − sin x sin y + y) dy = 0
(24) (2xy + y 2 ) dx + (2xy + x2 − 2x2 y 2 − 2xy 3 ) dy = 0
(25) y sin y dx + x(sin y − y cos y) dy = 0

In Exercises 26–32 find an integrating factor of the form µ(x, y) = P (x)Q(y) and
solve the given equation.
(26) y(1 + 5 ln |x|) dx + 4x ln |x| dy = 0
(27) (αy + γxy) dx + (βx + δxy) dy = 0
(28) (3x2 y 3 − y 2 + y) dx + (−xy + 2x) dy = 0
(29) 2y dx + 3(x2 + x2 y 3 ) dy = 0
(30) (a cos xy − y sin xy) dx + (b cos xy − x sin xy) dy = 0
(31) x4 y 4 dx + x5 y 3 dy = 0
(32) y(x cos x + 2 sin x) dx + x(y + 1) sin x dy = 0

In Exercises 33–36 find an integrating factor and solve the equation. Plot a direc-
tion field and some integral curves for the equation in the indicated rectangular
region.
(33) (x4 y 3 + y) dx + (x5 y 2 − x) dy = 0; {−1 ≤ x ≤ 1, −1 ≤ y ≤ 1}
(34) (3xy + 2y 2 + y) dx + (x2 + 2xy + x + 2y) dy = 0; {−2 ≤ x ≤ 2, −2 ≤ y ≤ 2}
(35) (12xy + 6y 3 ) dx + (9x2 + 10xy 2 ) dy = 0; {−2 ≤ x ≤ 2, −2 ≤ y ≤ 2}
(36) (3x2 y 2 + 2y) dx + 2x dy = 0; {−4 ≤ x ≤ 4, −4 ≤ y ≤ 4}
(37) Suppose a, b, c, and d are constants such that ad − bc 6= 0, and let m and n be
arbitrary real numbers. Show that
(axm y + by n+1 ) dx + (cxm+1 + dxy n ) dy = 0
has an integrating factor µ(x, y) = xα y β .
(38) Suppose M , N , Mx , and Ny are continuous for all (x, y), and µ = µ(x, y) is an
integrating factor for
M (x, y) dx + N (x, y) dy = 0. (A)
Assume that µx and µy are continuous for all (x, y), and suppose y = y(x) is a
differentiable function such that µ(x, y(x)) = 0 and µx (x, y(x)) 6= 0 for all x in
some interval I. Show that y is a solution of (A) on I.
(39) According to Theorem 19, the general solution of the linear nonhomogeneous
equation
y 0 + p(x)y = f (x) (A)
is  Z 
y = y1 (x) c + f (x)/y1 (x) dx , (B)

where y1 is any nontrivial solution of the complementary equation y 0 +p(x)y = 0.


In this exercise we obtain this conclusion in a different way.
(a) Rewrite (A) as
[p(x)y − f (x)] dx + dy = 0, (C)
R
and show that µ = ±e p(x) dx is an
R integrating factor for (C).
(b) Multiply (A) through by µ = ±e p(x) dx and verify that the resulting equa-
tion can be rewritten as
(µ(x)y)0 = µ(x)f (x).
49
Then integrate both sides of this equation and solve for y to show that the
general solution of (A) is
 Z 
1
y= c + f (x)µ(x) dx .
µ(x)
Why is this form of the general solution equivalent to (B)?
(40) Solve
x2 + y 2 dy
+ 2y = 0.
x+1 dx
(41) Solve
dy
y 2 + (xy + 1) = 0.
dx
(42) Solve the following exact equations, implicit general solutions will suffice:
a) (2xy + x2 ) dx + (x2 + y 2 + 1) dy = 0
dy
b) x5 + y 5 =0
dx
dy
c) ex + y 3 + 3xy 2 =0
dx
d) (x + y) cos(x) + sin(x) + sin(x)y 0 = 0
(43) Find the integrating factor for the following equations making them into exact
equations:
y
a) exy dx + exy dy = 0
x
ex + y 3
b) dx + 3x dy = 0
y2
2x + 2y 2
c) 4(y 2 + x) dx + dy = 0
y
d) 2 sin(y) dx + x cos(y) dy = 0
dy
(44) Suppose you have an equation of the form: f (x) + g(y) = 0.
dx
a) Show it is exact.
b) Find the form of the potential function in terms of f and g.
(45) Suppose that we have the equation f (x) dx − dy = 0.
a) Is this equation exact?
b) Find the general solution using a definite integral.
(46) Solve the following exact equations, implicit general solutions will suffice:
a) cos(x) + yexy + xexy y 0 = 0
b) (2x + y) dx + (x − 4y) dy = 0
dy
c) ex + ey =0
dx
d) (3x + 3y) dx + (3y 2 + 3x) dy = 0
2

a) exy + sin(x) = C, b) x2 + xy − 2y 2 = C, c) ex + ey = C, d)
3 3
x + 3xy + y = C.
(47) Find the integrating factor for the following equations making them into exact
equations:
1
a) dx + 3y dy = 0
y
b) dx − e−x−y dy = 0
50
cos(x) 1  x
c) 2
+ dx + 2 dy = 0
y y y
y2 
d) 2y + dx + (2y + x) dy = 0
x

Existence and Uniqueness of Solutions of Nonlinear Equations


Although there are methods for solving some nonlinear equations, it’s impossible to
find useful formulas for the solutions of most. Whether we’re looking for exact solutions
or numerical approximations, it’s useful to know conditions that imply the existence and
uniqueness of solutions of initial value problems for nonlinear equations. In this section
we state such a condition and illustrate it with examples.
Some terminology: an open rectangle R is a set of points (x, y) such that
a < x < b and c<y<d
We’ll denote this set by R : {a < x < b, c < y < d}. “Open” means that the boundary
rectangle isn’t included in R .
The next theorem gives sufficient conditions for existence and uniqueness of solutions
of initial value problems for first order nonlinear differential equations.
Theorem 40.

(1) If f is continuous on an open rectangle


R : {a < x < b, c < y < d}
that contains (x0 , y0 ) then the initial value problem
y 0 = f (x, y), y(x0 ) = y0 (100)
has at least one solution on some open subinterval of (a, b) that contains x0 .
(2) If both f and fy are continuous on R then (100) has a unique solution on some
open subinterval of (a, b) that contains x0 .
It’s important to understand exactly what Theorem 40 says.
• a) is an existence theorem . It guarantees that a solution exampleists on some
open interval that contains x0 , but provides no information on how to find the
solution, or to determine the open interval on which it exists. Moreover, a)
provides no information on the number of solutions that (100) may have. It
leaves open the possibility that (100) may have two or more solutions that differ
for values of x arbitrarily close to x0 . We will see in Example 46 that this can
happen.
• b) is a uniqueness theorem. It guarantees that (100) has a unique solution on
some open interval (a,b) that contains x0 . However, if (a, b) 6= (−∞, ∞), (100)
may have more than one solution on a larger interval that contains (a, b). For
example, it may happen that b < ∞ and all solutions have the same values
on (a, b), but two solutions y1 and y2 are defined on some interval (a, b1 ) with
b1 > b, and have different values for b < x < b1 ; thus, the graphs of the y1 and
y2 “branch off” in different directions at x = b. (See Example 47). In this case,
continuity implies that y1 (b) = y2 (b) (call their common value y), and y1 and y2
are both solutions of the initial value problem
y 0 = f (x, y), y(b) = y (101)
51
that differ on every open interval that contains b. Therefore f or fy must have
a discontinuity at some point in each open rectangle that contains (b, y), since
if this were not so, (101) would have a unique solution on some open interval
that contains b. We leave it to you to give a similar analysis of the case where
a > −∞.
Example 41. Consider the initial value problem
y 0 = x2 − y 2 /1 + x2 + y 2 , y(x0 ) = y0 . (102)
Since
x2 − y 2 2y(1 + 2x2 )
f (x, y) = and fy (x, y) = −
1 + x2 + y 2 (1 + x2 + y 2 )2
are continuous for all (x, y), Theorem 40 implies that if (x0 , y0 ) is arbitrary, then (102)
has a unique solution on some open interval that contains x0 .
··
^
Example 42. Consider the initial value problem
y 0 = x2 − y 2 /x2 + y 2 , y(x0 ) = y0 . (103)
Here
x2 − y 2 4x2 y
f (x, y) = and fy (x, y) = −
x2 + y 2 (x2 + y 2 )2
are continuous everywhere except at (0, 0). If (x0 , y0 ) 6= (0, 0), there’s an open rectangle
R that contains (x0 , y0 ) that does not contain (0, 0). Since f and fy are continuous on
R, Theorem 40 implies that if (x0 , y0 ) 6= (0, 0) then (103) has a unique solution on some
open interval that contains x0 .
··
^
Example 43. Consider the initial value problem
x+y
y0 = , y(x0 ) = y0 . (104)
x−y
Here
x+y 2x
f (x, y) = and fy (x, y) =
x−y (x − y)2
are continuous everywhere except on the line y = x. If y0 6= x0 , there’s an open rectangle
R that contains (x0 , y0 ) that does not intersect the line y = x. Since f and fy are
continuous on R, Theorem 40 implies that if y0 6= x0 , (104) has a unique solution on
some open interval that contains x0 .
··
^
Example 44. In Examplea 11 we saw that the solutions of
y 0 = 2xy 2 (105)
are
y ≡ 0 and y = −1/x2 + c,
where c is an arbitrary constant. In particular, this implies that no solution of (105)
other than y ≡ 0 can equal zero for any value of x. Show that Theorem 40b) implies this.
52
Solution. We’ll obtain a contradiction by assuming that (105) has a solution y1 that
equals zero for some value of x, but isn’t identically zero. If y1 has this property, there’s
a point x0 such that y1 (x0 ) = 0, but y1 (x) 6= 0 for some value of x in every open interval
that contains x0 . This means that the initial value problem
y 0 = 2xy 2 , y(x0 ) = 0 (106)
has two solutions y ≡ 0 and y = y1 that differ for some value of x on every open interval
that contains x0 . This contradicts Theorem 40(b), since in (105) the functions
f (x, y) = 2xy 2 and fy (x, y) = 4xy.
are both continuous for all (x, y), which implies that (106) has a unique solution on some
open interval that contains x0 .
··
^
Example 45. Consider the initial value problem
10
y 0 = xy 2/5 , y(x0 ) = y0 . (107)
3
(1) For what points (x0 , y0 ) does Theorem 40a) imply that (107) has a solution?
(2) For what points (x0 , y0 ) does Theorem 40b) imply that (107) has a unique solu-
tion on some open interval that contains x0 ?
Solution. Since
10 2/5
f (x, y) = xy
3
is continuous for all (x, y), Theorem 40 implies that (107) has a solution for every (x0 , y0 ).
Here
fy (x, y) = 4/3xy −3/5
is continuous for all (x, y) with y 6= 0. Therefore, if y0 6= 0 there’s an open rectangle on
which both f and fy are continuous, and Theorem 40 implies that (107) has a unique
solution on some open interval that contains x0 .
If y = 0 then fy (x, y) is undefined, and therefore discontinuous; hence, Theorem 40
does not apply to (107) if y0 = 0.
··
^
Example 46. Example 45 leaves open the possibility that the initial value problem
10
y 0 = xy 2/5 , y(0) = 0 (108)
3
has more than one solution on every open interval that contains x0 = 0. Show that this
is true.
Solution. By inspection, y ≡ 0 is a solution of the differential equation
y 0 = 10/3xy 2/5 . (109)
Since y ≡ 0 satisfies the initial condition y(0) = 0, it’s a solution of (108).
Now suppose y is a solution of (109) that isn’t identically zero. Separating variables
in (109) yields
10
y −2/5 y 0 = x
3
on any open interval where y has no zeros. Integrating this and rewriting the arbitrary
constant as 5c/3 yields
5 3/5 5 2
y = (x + c).
3 3
53
Therefore
y = (x2 + c)5/3 . (110)
Since we divided by y to separate variables in (109), our derivation of (110) is legiti-
mate only on open intervals where y has no zeros. However, (110) actually defines y for
all x, and differentiating (110) shows that
10 10
y 0 = x(x2 + c)2/3 = xy 2/5 , −∞ < x < ∞.
3 3
p p
Therefore (110) satisfies (109) on (−∞, ∞) even if c ≤ 0, so that y( |c|) = y(− |c|) = 0.
In particular, taking c = 0 in (110) yields
y = x10/3
as a second solution of (108). Both solutions are defined on (−∞, ∞), and they differ
on every open interval that contains x0 = 0. In fact, there are four distinct solutions of
(108) defined on (−∞, ∞) that differ from each other on every open interval that contains
x0 = 0. Can you identify the other two?
··
^
Example 47. From Example 45, the initial value problem
10
y 0 = xy 2/5 , y(0) = −1 (111)
3
has a unique solution on some open interval that contains x0 = 0. Find a solution and
determine the largest open interval (a, b) on which it’s unique.
Solution. Let y be any solution of (111). Because of the initial condition y(0) = −1
and the continuity of y, there’s an open interval I that contains x0 = 0 on which y has
no zeros, and is consequently of the form (110). Setting x = 0 and y = −1 in (110) yields
c = −1, so
y = (x2 − 1)5/3 (112)
for x in I. Therefore, every solution of (111) differs from zero and is given by (112) on
(−1, 1); that is, (112) is the unique solution of (111) on (−1, 1). This is the largest open
interval on which (111) has a unique solution. To see this, note that (112) is a solution
of (111) on (−∞, ∞). From exercise 2.2.24, there are infinitely many other solutions of
(111) that differ from (112) on every open interval larger than (−1, 1). One such solution
is ( 2
(x − 1)5/3 , −1 ≤ x ≤ 1,
y=
0, |x| > 1.
··
^
Example 48. From Example 45, the initial value problem
10
y 0 = xy 2/5 , y(0) = 1 (113)
3
has a unique solution on some open interval that contains x0 = 0. Find the solution and
determine the largest open interval on which it’s unique.
Solution. Let y be any solution of (113). Because of the initial condition y(0) = 1
and the continuity of y, there’s an open interval I that contains x0 = 0 on which y has
no zeros, and is consequently of the form (110). Setting x = 0 and y = 1 in (110) yields
c = 1, so
y = (x2 + 1)5/3 (114)
54
for x in I. Therefore every solution of (113) differs from zero and is given by (114) on
(−∞, ∞); that is, (114) is the unique solution of (113) on (−∞, ∞).
··
^
Exercises
In Exercises 1-13 find all (x0 , y0 ) for which Theorem 40 implies that the initial value
problem y 0 = f (x, y), y(x0 ) = y0 has a) a solution b) a unique solution on some open
interval that contains x0 .

0x2 + y 2 (7) y 0 = ln(1 + x2 + y 2 )


(1) y = 2x + 3y
sin x (8) y 0 =
x − 4y
ex + y
(2) y 0 = (9) y 0 = (x2 + y 2 )1/2
x2 + y 2
0 (10) y 0 = x(y 2 − 1)2/3
(3) y = tan xy
x2 + y 2 (11) y 0 = (x2 + y 2 )2
(4) y0 = (12) y 0 = (x + y)1/2
ln xy tan y
(5) 0
y = (x2 + y 2 )y 1/3 (13) y 0 =
x−1
(6) y 0 = 2xy

(14) Apply Theorem 40 to the initial value problem


y 0 + p(x)y = q(x), y(x0 ) = y0
for a linear equation, and compare the conclusions that can be drawn from it to
those that follow from Theorem 2.1.2.
(15) (a) Verify that the function
( 2
(x − 1)5/3 , −1 < x < 1,
y=
0, |x| ≥ 1,
is a solution of the initial value problem
10
y 0 = xy 2/5 , y(0) = −1
3
on (−∞, ∞).
(b) Verify that if εi = 0 or 1 for i = 1, 2 and a, b > 1, then the function
ε1 (x2 − a2 )5/3 , −∞ < x < −a,




0, −a ≤ x ≤ −1,





y= (x2 − 1)5/3 , −1 < x < 1,





 0, 1 ≤ x ≤ b,

ε2 (x2 − b2 )5/3 , b < x < ∞,

is a solution of the initial value problem of a) on (−∞, ∞).


(16) Use the ideas developed in Exercise 15 to find infinitely many solutions of the
initial value problem
y 0 = y 2/5 , y(0) = 1
on (−∞, ∞).
(17) Consider the initial value problem
y 0 = 3x(y − 1)1/3 , y(x0 ) = y0 . (A)
55
(a) For what points (x0 , y0 ) does Theorem 40 imply that (A) has a solution?
(b) For what points (x0 , y0 ) does Theorem 40 imply that (A) has a unique
solution on some open interval that contains x0 ?
(18) Find nine solutions of the initial value problem
y 0 = 3x(y − 1)1/3 , y(0) = 1
that are all defined on (−∞, ∞) and differ from each other for values of x in
every open interval that contains x0 = 0.
(19) From Theorem 40, the initial value problem
y 0 = 3x(y − 1)1/3 , y(0) = 9
has a unique solution on an open interval that contains x0 = 0. Find the solution
and determine the largest open interval on which it’s unique.
(20) (a) From Theorem 40, the initial value problem
y 0 = 3x(y − 1)1/3 , y(3) = −7 (A)
has a unique solution on some open interval that contains x0 = 3. Determine
the largest such open interval, and find the solution on this interval.
(b) Find infinitely many solutions of (A), all defined on (−∞, ∞).
(21) Prove:
(a) If
f (x, y0 ) = 0, a < x < b, (A)
and x0 is in (a, b), then y ≡ y0 is a solution of
y 0 = f (x, y), y(x0 ) = y0
on (a, b).
(b) If f and fy are continuous on an open rectangle that contains (x0 , y0 ) and
(A) holds, no solution of y 0 = f (x, y) other than y ≡ y0 can equal y0 at any
point in (a, b).

Transformation of Nonlinear Equations into Separable Equations


We found that the solutions of a linear nonhomogeneous equation
y 0 + p(x)y = f (x)
are of the form y = uy1 , where y1 is a nontrivial solution of the complementary equation
y 0 + p(x)y = 0 (115)
and u is a solution of
u0 y1 (x) = f (x).
Note that this last equation is separable, since it can be rewritten as
f (x)
u0 = .
y1 (x)
We will consider nonlinear differential equations that are not separable to begin with, but
can be solved in a similar fashion by writing their solutions in the form y = uy1 , where
y1 is a suitably chosen known function and u satisfies a separable equation. We’llsay in
this case that we transformed the given equation into a separable equation.
56
Bernoulli Equations. A Bernoulli equation is an equation of the form
y 0 + p(x)y = f (x)y r , (116)
where r can be any real number other than 0 or 1. (Note that (116) is linear if and only
if r = 0 or r = 1.) We can transform (116) into a separable equation by variation of
parameters: if y1 is a nontrivial solution of (115), substituting y = uy1 into (116) yields
u0 y1 + u(y10 + p(x)y1 ) = f (x)(uy1 )r ,
which is equivalent to the separable equation
u0 y1 (x) = f (x) (y1 (x))r ur or u0 /ur = f (x) (y1 (x))r−1 ,
since y10 + p(x)y1 = 0.
Example 49. Solve the Bernoulli equation
y 0 − y = xy 2 . (117)
Solution. Since y1 = ex is a solution of y 0 − y = 0, we look for solutions of (117) in
the form y = uex , where
u0 ex = xu2 e2x or, equivalently, u0 = xu2 ex .
Separating variables yields
u0
2
= xex ,
u
and integrating yields
−1
= (x − 1)ex + c.
u
Hence,
1
u=−
(x − 1)ex + c
and
1
y=− .
x − 1 + ce−x
Other Nonlinear Equations That Can be Transformed Into Separable
Equations. We’ve seen that the nonlinear Bernoulli equation can be transformed into a
separable equation by the substitution y = uy1 if y1 is suitably chosen. Now let’s discover
a sufficient condition for a nonlinear first order differential equation
y 0 = f (x, y) (118)
to be transformable into a separable equation in the same way. Substituting y = uy1 into
(118) yields
u0 y1 (x) + uy10 (x) = f (x, uy1 (x)),
which is equivalent to
u0 y1 (x) = f (x, uy1 (x)) − uy10 (x). (119)
If
f (x, uy1 (x)) = q(u)y10 (x)
for some function q, then (119) becomes
u0 y1 (x) = (q(u) − u)y10 (x), (120)
57
which is separable. After checking for constant solutions u ≡ u0 such that q(u0 ) = u0 ,
we can separate variables to obtain
u0 /q(u) − u = y10 (x)/y1 (x).

Homogeneous equations. Another type of equations we can solve by substitution


are the so-called homogeneous equations. Suppose that we can write the differential
equation as y
y0 = F .
x
Here we try the substitutions
y
v= and therefore y 0 = v + xv 0 .
x
We note that the equation is transformed into
v0 1
v + xv 0 = F (v) or xv 0 = F (v) − v or = .
F (v) − v x
Hence an implicit solution is
Z
1
dv = ln |x| + C.
F (v) − v
Example 50. Solve
x2 y 0 = y 2 + xy, y(1) = 1.
 y 2 y y
We put the equation into the form y 0 = + . We substitute v = to get the
x x x
separable equation
xv 0 = v 2 + v − v = v 2 ,
which has a solution
Z
1
dv = ln |x| + C,
v2
−1
= ln |x| + C,
v
−1
v= .
ln |x| + C
We unsubstitute
y −1
= ,
x ln |x| + C
−x
y= .
ln |x| + C
We want y(1) = 1, so
−1 −1
1 = y(1) = = .
ln |1| + C C
Thus C = −1 and the solution we are looking for is
−x
y= .
ln |x| − 1
··
^
58
Example 51. Solve
y + xe−y/x
y0 = . (121)
x
Solution. Substituting y = ux into (121) yields
ux + xe−ux/x
u0 x + u = = u + e−u .
x
Simplifying and separating variables yields
1
eu u0 = .
x
u
Integrating yields e = ln |x| + c. Therefore, u = ln(ln |x| + c) and
y = ux = x ln(ln |x| + c) .
··
^
Example 52.

(1) Solve
x2 y 0 = y 2 + xy − x2 . (122)
(2) Solve the initial value problem
x2 y 0 = y 2 + xy − x2 , y(1) = 2. (123)
Solution. We first find solutions of (122) on open intervals that don’t contain x = 0.
We can rewrite (122) as
y 2 + xy − x2
y0 =
x2
for x in any such interval. Substituting y = ux yields
(ux)2 + x(ux) − x2
u0 x + u = = u2 + u − 1,
x2
so
u0 x = u2 − 1. (124)
By inspection this equation has the constant solutions u ≡ 1 and u ≡ −1. Therefore
y = x and y = −x are solutions of (122). If u is a solution of (124) that doesn’t assume
the values ±1 on some interval, separating variables yields
u0 1
2
= ,
u −1 x
or, after a partial fraction examplepansion,
 
1 1
1/2 − u0 = 1/x.
u−1 u+1
Multiplying by 2 and integrating yields

u − 1
ln
= 2 ln |x| + k,
u + 1
or
u − 1 k 2
u + 1 = e x ,

59
which holds if
u−1
= cx2 (125)
u+1
where c is an arbitrary constant. Solving for u yields
1 + cx2
u= .
1 − cx2
Therefore
x(1 + cx2 )
y = ux = (126)
1 − cx2
is a solution of (123) for any choice of the constant c. Setting c = 0 in (126) yields the
solution y = x. However, the solution y = −x can’t be obtained from (126). Thus, the
solutions of (122) on intervals that don’t contain x = 0 are y = −x and functions of the
form (126).
The situation is more complicated if x = 0 is the open interval. First, note that
y = −x satisfies (122) on (−∞, ∞). If c1 and c2 are arbitrary constants, the function

x(1 + c1 x2 )
, a < x < 0,


− 2

y= 1 c 1 x (127)
x(1 + c2 x2 )
, 0 ≤ x < b,


1 − c2 x 2

is a solution of (122) on (a, b), where


 √  √
−1/ c1 if c1 > 0, 1/ c2 if c2 > 0,
a= and b=
−∞ if c1 ≤ 0, ∞ if c2 ≤ 0.
We leave it to you to verify this. To do so, note that if y is any function of the form (126)
then y(0) = 0 and y 0 (0) = 1.
We could obtain c by imposing the initial condition y(1) = 2 in (126), and then solving
for c. However, it’s easier to use (125). Since u = y/x, the initial condition y(1) = 2
implies that u(1) = 2. Substituting this into (125) yields c = 1/3. Hence, the solution of
(123) is
x(1 + x2 /3)
y= .
1 − x2 /3
√ √
The interval of validity of this solution is √(− 3, 3). However, the largest interval on
which (123) has a unique solution is (0, 3). To see this, note from (127) that any
function of the form 
x(1 + cx2 )
, a < x ≤ 0,


1 − cx2

y= √ (128)
x(1 + x2 /3)
, 0 ≤ x < 3,


1 − x2 /3

√ √
is a solution of (123) on (a, 3), where a = −1/ c if c > 0 or a = −∞ if c ≤ 0.
··
^

In the last two examples we were able to solve the given equations explicitly. However,
this isn’t always possible, as you’ll see in the exercises.
Example 53. Solve
xy 0 + y(x + 1) + xy 5 = 0, y(1) = 1.
60
First, the equation is Bernoulli (p(x) = (x + 1)/x and q(x) = −1). We substitute
v = y 1−5 = y −4 , v 0 = −4y −5 y 0 .
 
−1
In other words, y 5 v 0 = y 0 . So
4
xy 0 + y(x + 1) + xy 5 = 0,
−xy 5 0
v + y(x + 1) + xy 5 = 0,
4
−x 0
v + y −4 (x + 1) + x = 0,
4
−x 0
v + v(x + 1) + x = 0,
4
and finally
4(x + 1)
v0 −
v = 4.
x
The equation is now linear. We can use the integrating factor method. Let us assume
that x > 0 so |x| = x. This assumption is OK, as our initial condition is x = 1. Let us
compute the integrating factor.
Z x
e−4x+4

Rx −4(s + 1)
e1 p(s) ds
= ds = e−4x−4 ln(x)+4 = e−4x+4 x−4 = ,
1 s x4
Rx
e− 1 p(s) ds
= e4x+4 ln(x)−4 = e4x−4 x4 .

Rx
Z x R 
t
− p(s) ds p(s) ds
v(x) = e 1 e 1 4 dt + 1
1
x
e−4t+4
Z 
4x−4 4
=e x 4 dt + 1 .
1 t4
The integral in this examplepression is not possible to find in closed form. As we said
before, it is perfectly fine to have a definite integral in our solution. Now unsubstitute
 Z x −4t+4 
−4 4x−4 4 e
y =e x 4 dt + 1 ,
1 t4

e−x+1
y= Rx 1/4 .
e−4t+4
x 4 1 t4
dt + 1
··
^
Exercises
In Exercises 1–4 solve the given Bernoulli equation.
(1) y 0 + y = y 2
x2
(2) 7xy 0 − 2y = − 6
y
(3) x2 y 0 + 2y = 2e1/x y 1/2
1
(4) (1 + x2 )y 0 + 2xy =
(1 + x2 )y
In Exercises 5 and 6 find all solutions. Also, plot a direction field and some
integral curves on the indicated rectangular region.
61
(5) y 0 − xy = x3 y 3 ; {−3 ≤ x ≤ 3, 2 ≤ y ≥ 2}
1+x
(6) y 0 − y = y 4 ; {−2 ≤ x ≤ 2, −2 ≤ y ≤ 2}
3x
In Exercises 7–11 solve the√ initial value problem.
0 3
(7) y − 2y = xy , y(0) = 2 2
(8) y 0 − xy = xy 3/2 , y(1) = 4
(9) xy 0 + y = x4 y 4 , y(1) = 1/2
(10) y 0 − 2y = 2y 1/2 , y(0) = 1
48x
(11) y 0 − 4y = 2 , y(0) = 1
y
In Exercises 12 and 13 solve √the initial value problem and graph the solution.
(12) x2 y 0 + 2xy = y 3 , y(1) = 1/ 2
(13) y 0 − y = xy 1/2 , y(0) = 4
(14) You may have noticed that the logistic equation
P 0 = aP (1 − αP )
from Verhulst’s model for population growth can be written in Bernoulli form as
P 0 − aP = −aαP 2 .
This isn’t particularly interesting, since the logistic equation is separable, and
therefore solvable by the method studied in Section 2.2. So let’s consider a more
complicated model, where a is a positive constant and α is a positive continuous
function of t on [0, ∞). The equation for this model is
P 0 − aP = −aα(t)P 2 ,
a non-separable Bernoulli equation.
(a) Assuming that P (0) = P0 > 0, find P for t > 0.
(b) Verify that your result reduces to the known results for the Malthusian
model where α = 0, and the Verhulst model where α is a nonzero constant.
(c) Assuming that
Z t
−at
lim e α(τ )eaτ dτ = L
t→∞ 0

exampleists (finite or infinite), find lim P (t).


t→∞
In Exercises 15–18 solve the equation explicitly.
y+x
(15) y0 =
x
2
y + 2xy
(16) y0 =
x2
3 0
(17) xy y = y 4 + x4
y
(18) y 0 = + sec y/x
x
In Exercises 19-21 solve the equation exampleplicitly. Also, plot a direction field
and some integral curves on the indicated rectangular region.
(19) x2 y 0 = xy + x2 + y 2 ; {−8 ≤ x ≤ 8, −8 ≤ y ≤ 8}
(20) xyy 0 = x2 + 2y 2 ; {−4 ≤ x ≤ 4, −4 ≤ y ≤ 4}
62
2
0 2y 2 + x2 e−(y/x)
(21) y = ; {−8 ≤ x ≤ 8, −8 ≤ y ≤ 8}
2xy
In Exercises 22–27 solve the initial value problem.

xy + y 2 y 2 − 3xy − 5x2
(22) y 0 = , y(−1) = 2 (25) y 0 = , y(1) = −1
x2 x2
3 3 2 0 2 2
x +y (26) x y = 2x + y + 4xy, y(1) √=1
(23) y 0 = , y(1) = 3
xy 2 (27) xyy 0 = 3x2 + 4y 2 , y(1) = 3
(24) xyy 0 + x2 + y 2 = 0, y(1) = 2

In Exercises 28–34 solve the given homogeneous equation implicitly.


x+y
(28) y 0 =
x−y

(29) (y 0 x − y)(ln |y| − ln |x|) = x


y 3 + 2xy 2 + x2 y + x3
(30) y 0 =
x(y + x)2
x + 2y
(31) y 0 =
2x + y
0 y
(32) y =
y − 2x
xy 2 + 2y 3
(33) y 0 = 3
x + x2 y + xy 2
x3 + x2 y + 3y 3
(34) y 0 =
x3 + 3xy 2
(35) (a) Find a solution of the initial value problem
x2 y 0 = y 2 + xy − 4x2 , y(−1) = 0 (A)
on the interval (−∞, 0). Verify that this solution is actually valid on
(−∞, ∞).
(b) Use Theorem 40 to show that (A) has a unique solution on (−∞, 0).
(c) Plot a direction field for the differential equation in (A) on a square
{−r ≤ x ≤ r, −r ≤ y ≤ r},
where r is any positive number.
(d) Graph other solutions of (A) that are defined on (−∞, ∞).
(e) Graph other solutions of (A) that are defined only on intervals of the form
(−∞, a), where is a finite positive number.
(36) (a) Solve the equation
xyy 0 = x2 − xy + y 2 (A)
implicitly.
(b) Plot a direction field for (A) on a square
{0 ≤ x ≤ r, 0 ≤ y ≤ r}
where r is any positive number.
(c) Let K be a positive integer. (You may have to try several choices for K.)
Graph solutions of the initial value problems
xyy 0 = x2 − xy + y 2 , y(r/2) = kr/K,
63
for k = 1, 2, . . . , K. Based on your observations, find conditions on the
positive numbers x0 and y0 such that the initial value problem
xyy 0 = x2 − xy + y 2 , y(x0 ) = y0 , (B)
has a unique solution (i) on (0, ∞) or (ii) only on an interval (a, ∞), where
a > 0?
(d) What can you say about the graph of the solution of (B) as x → ∞? (Again,
assume that x0 > 0 and y0 > 0.)
(37) (a) Solve the equation
y 0 = 2y 2 − xy + 2x2 /xy + 2x2 (A)
implicitly.
(b) Plot a direction field for (A) on a square
{−r ≤ x ≤ r, −r ≤ y ≤ r}
where r is any positive number. By graphing solutions of (A), determine
necessary and sufficient conditions on (x0 , y0 ) such that (A) has a solution
on (i) (−∞, 0) or (ii) (0, ∞) such that y(x0 ) = y0 .
(38) Follow the instructions of exampleercise 37 for the equation
xy + x2 + y 2
y0 = .
xy
(39) Pick any nonlinear homogeneous equation y 0 = q(y/x) you like, and plot direction
fields on the square {−r ≤ x ≤ r, −r ≤ y ≤ r}, where r > 0. What happens to
the direction field as you vary r? Why?
(40) Prove: If ad − bc 6= 0, the equation
ax + by + α
y0 =
cx + dy + β
can be transformed into the homogeneous nonlinear equation
dY aX + bY
=
dX cX + dY
by the substitution x = X − X0 , y = Y − Y0 , where X0 and Y0 are suitably
chosen constants.

In Exercises 41-43 use a method suggested by Exercise 40 to solve the given


equation implicitly.
−6x + y − 3
(41) y 0 =
2x − y − 1
2x +y+1
(42) y 0 =
x + 2y − 4
−x + 3y − 14
(43) y 0 =
x+y−2
In Exercises 44–51 find a function y1 such that the substitution y = uy1 trans-
forms the given equation into a separable equation of the form (120). Then solve
the given equation exampleplicitly.

(44) 3xy 2 y 0 = y 3 + x
(45) xyy 0 = 3x6 + 6y 2
64
(46) x3 y 0 = 2(y 2 + x2 y − x4 ) (47) y 0 = y 2 e−x + 4y + 2ex

y 2 + y tan x + tan2 x (49) x(ln x)2 y 0 = −4(ln x)2 +y ln x+y 2


(48) y 0 =
sin2 x
√ √
(50) 2x(y + 2 x)y 0 = (y + x)2 (51) (y + ex )y 0 = 2x(y 2 + yex + e2x )
2 2 2

(52) Solve the initial value problem


2 3x2 y 2 + 6xy + 2
y0 + y = , y(2) = 2.
x x2 (2xy + 3)
(53) Solve the initial value problem
0 3 3x4 y 2 + 10x2 y + 6
y + y= , y(1) = 1.
x x3 (2x2 y + 5)
(54) Prove: If y is a solution of a homogeneous nonlinear equation y 0 = q(y/x), so is
y1 = y(ax)/a, where a is any nonzero constant.
(55) A generalized Riccati equation is of the form
y 0 = P (x) + Q(x)y + R(x)y 2 . (A)
(If R ≡ −1, (A) is a Riccati equation.) Let y1 be a known solution and y an
arbitrary solution of (A). Let z = y − y1 . Show that z is a solution of a Bernoulli
equation with n = 2.

In Exercises 56–59, given that y1 is a solution of the given equation, use the
method suggested by exercise 55 to find other solutions.
(56) y 0 = 1 + x − (1 + 2x)y + xy 2 ; y1 = 1
(57) y 0 = e2x + (1 − 2ex )y + y 2 ; y1 = ex
(58) xy 0 = 2 − x + (2x − 2)y − xy 2 ; y1 = 1
(59) xy 0 = x3 + (1 − 2x2 )y + xy 2 ; y1 = x
(60) Solve y 0 + y(x2 − 1) + xy 6 = 0, with y(1) = 1.
(61) Solve 2yy 0 + 1 = y 2 + x, with y(0) = 1.
(62) Solve y 0 + xy = yp4 , with y(0) = 1.
(63) Solve yy 0 + x = x2 + y 2 .
(64) Solve y 0 = (x + y − 1)2 .
x2 − y 2
(65) Solve y 0 = , with y(1) = 2.
xy
(66) Solve xy 0 + y + y 2 = 0, y(1) = 2.
(67) Solve xy 0 + y + x = 0, y(1) = 1.
(68) Solve y 2 y 0 = y 3 − 3x, y(0) = 2.
2 2
(69) Solve 2yy 0 = ey −x + 2x.
(70) Solve x2 y 0 = y 2 + xy, y(1) = 1.

Applications of First Order Equations: Applications to Curves


One-Parameter Families of Curves. We begin with two examples of families of
curves generated by varying a parameter over a set of real numbers.
Example 54. For each value of the parameter c, the equation
y − cx2 = 0 (129)
65
6 0, the curve is a parabola through the origin,
defines a curve in the xy-plane. If c =
opening upward if c > 0 or downward if c < 0. If c = 0, the curve is the x axis
Example 55. For each value of the parameter c the equation
y =x+c (130)
defines a line with slope 1
Definition. An equation that can be written in the form
H(x, y, c) = 0 (131)
is said to define a one-parameter family of curves if, for each value of c in in some
nonempty set of real numbers, the set of points (x, y) that satisfy (131) forms a curve in
the xy-plane.
Equations (129) and (130) define one–parameter families of curves. (Although (130)
isn’t in the form (131), it can be written in this form as y − x − c = 0.)
Example 56. If c > 0, the graph of the equation
x2 + y 2 − c = 0 (132)

is a circle with center at (0, 0) and radius c. If c = 0, the graph is the single point
(0, 0). (We don’t regard a single point as a curve.) If c < 0, the equation has no graph.
Hence, (132) defines a one–parameter family of curves for positive values of c. This family
consists of all circles centered at (0, 0) (Figure ??).
Example 57. The equation
x 2 + y 2 + c2 = 0
does not define a one-parameter family of curves, since no (x, y) satisfies the equation if
c 6= 0, and only the single point (0, 0) satisfies it if c = 0.
Recall from Section 1.2 that the graph of a solution of a differential equation is
called an integral curve of the equation. Solving a first order differential equation usually
produces a one–parameter family of integral curves of the equation. Here we are interested
in the converse problem: given a one–parameter family of curves, is there a first order
differential equation for which every member of the family is an integral curve. This
suggests the nexamplet definition.
Definition. If every curve in a one-parameter family defined by the equation
H(x, y, c) = 0 (133)
is an integral curve of the first order differential equation
F (x, y, y 0 ) = 0, (134)
then (134) is said to be a differential equation for the family.
To find a differential equation for a one–parameter family we differentiate its defining
equation (133) implicitly with respect to x, to obtain
Hx (x, y, c) + Hy (x, y, c)y 0 = 0. (135)
If this equation doesn’t, then it’s a differential equation for the family. If it does contain
c, it may be possible to obtain a differential equation for the family by eliminating c
between (133) and (135).
66
Example 58. Find a differential equation for the family of curves defined by
y = cx2 . (136)
Solution. Differentiating (136) with respect to x yields
y 0 = 2cx.
Therefore c = y 0 /2x, and substituting this into (136) yields
y = xy 0 /2
as a differential equation for the family of curves defined by (136). The graph of any
function of the form y = cx2 is an integral curve of this equation.
The nexamplet exampleample shows that members of a given family of curves may
be obtained by joining integral curves for more than one differential equation.
Example 59.
(1) Try to find a differential equation for the family of lines tangent to the parabola
y = x2 .
(2) Find two tangent lines to the parabola y = x2 that pass through (2, 3), and find
the points of tangency.
Solution. The equation of the line through a given point (x0 , y0 ) with slope m is
y = y0 + m(x − x0 ). (137)
If (x0 , y0 ) is on the parabola, then y0 = x20 and the slope of the tangent line through
(x0 , x20 ) is m = 2x0 ; hence, (137) becomes
y = x20 + 2x0 (x − x0 ),
or, equivalently,
y = −x20 + 2x0 x. (138)
Here x0 plays the role of the constant c in Definition 1; that is, varying x0 over (−∞, ∞)
produces the family of tangent lines to the parabola y = x2 .
Differentiating (138) with respect to x yields y 0 = 2x0 .. We can examplepress x0 in
terms of x and y by rewriting (138) as
x20 − 2x0 x + y = 0
and using the quadratic formula to obtain
p
x0 = x ± x2 − y. (139)
We must choose the plus sign in (139) if x < x0 and the minus sign if x > x0 ; thus,
 p 
x0 = x + x2 − y if x < x0
and  p 
x0 = x − x2 − y if x > x0 .
Since y 0 = 2x0 , this implies that
p  
0 2
y =2 x+ x −y , if x < x0 (140)
and  p 
y 0 = 2 x − x2 − y , if x > x0 . (141)
Neither (140) nor (141) is a differential equation for the family of tangent lines to the
parabola y = x2 . However, if each tangent line is regarded as consisting of two tangent
67
half lines joined at the point of tangency, (140) is a differential equation for the family
of tangent half lines on which x is less than the abscissa of the point of tangency (Fig-
ure ??a)), while (141) is a differential equation for the family of tangent half lines on
which x is greater than this abscissa. The parabola y = x2 is also an integral curve of
both (140) and (141).
From (138) the point (x, y) = (2, 3) is on the tangent line through (x0 , x20 ) if and only
if
3 = −x20 + 4x0 ,
which is equivalent to
x20 − 4x0 + 3 = (x0 − 3)(x0 − 1) = 0.
Letting x0 = 3 in (138) shows that (2, 3) is on the line
y = −9 + 6x,
which is tangent to the parabola at (x0 , x20 ) = (3, 9), as shown in Figure ??
Letting x0 = 1 in (138) shows that (2, 3) is on the line
y = −1 + 2x,
which is tangent to the parabola at (x0 , x20 ) = (1, 1).
Geometric Problems. We now consider some geometric problems that can be
solved by means of differential equations.
Example 60. Find curves y = y(x) such that every point (x0 , y(x0 )) on the curve is
the midpoint of the line segment with endpoints on the coordinate axes and tangent to
the curve at (x0 , y(x0 ))
Solution. The equation of the line tangent to the curve at P = (x0 , y(x0 ) is
y = y(x0 ) + y 0 (x0 )(x − x0 ).
If we denote the x and y intercepts of the tangent line by xI and yI , then
0 = y(x0 ) + y 0 (x0 )(xI − x0 ) (142)
and
yI = y(x0 ) − y 0 (x0 )x0 . (143)
P is the midpoint of the line segment connecting (xI , 0) and (0, yI ) if and only if xI = 2x0
and yI = 2y(x0 ). Substituting the first of these conditions into (142) or the second into
(143) yields
y(x0 ) + y 0 (x0 )x0 = 0.
Since x0 is arbitrary we drop the subscript and conclude that y = y(x) satisfies
y + xy 0 = 0,
which can be rewritten as
(xy)0 = 0.
Integrating yields xy = c, or
y = c/x.
If c = 0 this curve is the line y = 0, which does not satisfy the geometric requirements
imposed by the problem; thus, c 6= 0, and the solutions define a family of hyperbolas .
Example 61. Find curves y = y(x) such that the tangent line to the curve at any
point (x0 , y(x0 )) intersects the x-axis at (x20 , 0).
68
Solution. The equation of the line tangent to the curve at (x0 , y(x0 )) is
y = y(x0 ) + y 0 (x0 )(x − x0 ).
Since (x20 , 0) is on the tangent line,
0 = y(x0 ) + y 0 (x0 )(x20 − x0 ).
Since x0 is arbitrary we drop the subscript and conclude that y = y(x) satisfies
y + y 0 (x2 − x) = 0.
Therefor
y 0 /y = −1/x2 − x = −1/x(x − 1) = 1/x − 1/x − 1,
so
ln |y| = ln |x| − ln |x − 1| + k = ln |x/x − 1| + k,
and
y = cx/x − 1.
If c = 0, the graph of this function is the x-axis. If c 6= 0, it’s a hyperbola with vertical
asymptote x = 1 and horizontal asymptote y = c. Figure ?? shows the graphs for c 6= 0.
Orthogonal Trajectories. Two curves C1 and C2 are said to be orthogonal at a
point of intersection (x0 , y0 ) if they have perpendicular tangents at (x0 , y0 ). A curve is
said to be an orthogonal trajectory of a given family of curves if it’s orthogonal to every
curve in the family. For instance, every line through the origin is an orthogonal trajectory
of the family of circles centered at the origin. Conversely, any such circle is an orthogonal
trajectory of the family of lines through the origin.
Orthogonal trajectories occur in many physical applications. For example, if u =
u(x, y) is the temperature at a point (x, y), the curves defined by
u(x, y) = c (144)
are called isothermal curves. The orthogonal trajectories of this family are called heat-
flow lines, because at any given point the direction of maximum heat flow is perpendicular
to the isothermal through the point. If u represents the potential energy of an object
moving under a force that depends upon (x, y), the curves (144) are called equipotentials,
and the orthogonal trajectories are called lines of force.
From analytic geometry we know that two nonvertical lines L1 and L2 with slopes
m1 and m2 , respectively, are perpendicular if and only if m2 = −1/m1 ; therefore, the
integral curves of the differential equation
1
y0 = −
f (x, y)
are orthogonal trajectories of the integral curves of the differential equation
y 0 = f (x, y),
because at any point (x0 , y0 ) where curves from the two families intersect the slopes of
the respective tangent lines are
1
m1 = f (x0 , y0 ) and m2 = − .
f (x0 , y0 )
This suggests a method for finding orthogonal trajectories of a family of integral
curves of a first order equation.
Finding Orthogonal Trajectories
69
Step 1. Find a differential equation
y 0 = f (x, y)
for the given family.
Step 2. Solve the differential equation
1
y0 = −
f (x, y)
to find the orthogonal trajectories.
Example 62. Find the orthogonal trajectories of the family of circles
x 2 + y 2 = c2 (c > 0). (145)
Solution. To find a differential equation for the family of circles we differentiate
(145) implicitly with respect to x to obtain
2x + 2yy 0 = 0,
or
y 0 = −x/y.
Therefore the integral curves of
y 0 = y/x
are orthogonal trajectories of the given family. We leave it to you to verify that the
general solution of this equation is
y = kx,
where k is an arbitrary constant. This is the equation of a nonvertical line through
(0, 0). The y axis is also an orthogonal trajectory of the given family. Therefore every
line through the origin is an orthogonal trajectory of the given family (145). This is
consistent with the theorem of plane geometry which states that a diameter of a circle
and a tangent line to the circle at the end of the diameter are perpendicular.
Example 63. Find the orthogonal trajectories of the family of hyperbolas
xy = c (c 6= 0) (146)
Solution. Differentiating (146) implicitly with respect to x yields
y + xy 0 = 0,
or
y
y0 = − ;
x
thus, the integral curves of
x
y0 =
y
are orthogonal trajectories of the given family. Separating variables yields
y0y = x
and integrating yields
y 2 − x2 = k,
which is the equation of a hyperbola if k 6= 0, or of the lines y = x and y = −x if k = 0
(Figure ??).
70
Example 64. Find the orthogonal trajectories of the family of circles defined by
(x − c)2 + y 2 = c2 (c 6= 0). (147)
These circles are centered on the x-axis and tangent to the y-axis (Figure ??a) ).
Solution. Multiplying out the left side of (147) yields
x2 − 2cx + y 2 = 0, (148)
and differentiating this implicitly with respect to x yields
2(x − c) + 2yy 0 = 0. (149)
From (148),
c = x2 + y 2 /2x,
so
x2 + y 2 x2 − y 2
x−c=x− = .
2x 2x
Substituting this into (149) and solving for y 0 yields
y 2 − x2
y0 = . (150)
2xy
The curves defined by (147) are integral curves of (150), and the integral curves of
2xy
y0 = 2
x − y2
are orthogonal trajectories of the family (147). This is a homogeneous nonlinear equation,
which we studied in Section 2.4. Substituting y = ux yields
2x(ux) 2u
u0 x + u = 2 2
= ,
x − (ux) 1 − u2
so
2u u(u2 + 1)
u0 x = − u = ,
1 − u2 1 − u2
Separating variables yields
1 − u2 0 1
u = ,
u(u2 + 1) x
or, equivalently,  
1 2u 1
− 2 u0 = .
u u +1 x
Therefore
ln |u| − ln(u2 + 1) = ln |x| + k.
By substituting u = y/x, we see that
ln |y| − ln |x| − ln(x2 + y 2 ) + ln(x2 ) = ln |x| + k,
which, since ln(x2 ) = 2 ln |x|, is equivalent to
ln |y| − ln(x2 + y 2 ) = k,
or
|y| = ek (x2 + y 2 ).
To see what these curves are we rewrite this equation as
x2 + |y|2 − e−k |y| = 0
71
and complete the square to obtain
x2 + (|y| − e−k /2)2 = (e−k /2)2 .
This can be rewritten as
x2 + (y − h)2 = h2 ,
where  −k
 e

if y ≥ 0,
h= 2 −k
 −e

if y ≤ 0.
2
Thus, the orthogonal trajectories are circles centered on the y axis and tangent to the x
axis. The circles for which h > 0 are above the x-axis, while those for which h < 0 are
below.
Exercises
In Exercises 1–8 find a first order differential equation for the given family of curves.
(1) y(x2 + y 2 ) = c
(2) exy = cy
(3) ln |xy| = c(x2 + y 2 )
(4) y = x1/2 + cx
2 2
(5) y = ex + ce−x
c
(6) y = x3 +
x
(7) y = sin x + cex
(8) y = ex + c(1 + x2 )
(9) Show that the family of circles
(x − x0 )2 + y 2 = 1, −∞ < x0 < ∞,
can be obtained by joining integral curves of two first order differential equations.
More specifically, find differential equations for the families of semicircles
(x − x0 )2 + y 2 = 1, x0 < x < x0 + 1, −∞ < x0 < ∞,
(x − x0 )2 + y 2 = 1, x0 − 1 < x < x0 , −∞ < x0 < ∞.
(10) Suppose f and g are differentiable for all x. Find a differential equation for the
family of functions y = f + cg (c=constant).

In Exercises 11–13 find a first order differential equation for the given family of
curves.
(11) Lines through a given point (x0 , y0 ).
(12) Circles through (−1, 0) and (1, 0).
(13) Circles through (0, 0) and (0, 2).
(14) Use the method exampleample 59(a) to find the equations of lines through the
given points tangent to the parabola y = x2 . Also, find the points of tangency.
(a) (5, 9) (b) (6, 11) (c) (−6, 20) (d) (−3, 5)
(15) (a) Show that the equation of the line tangent to the circle
x2 + y 2 = 1 (A)
at a point (x0 , y0 ) on the circle is
1 − x0 x
y= if x0 6= ±1. (B)
y0
72
(b) Show that if y 0 is the slope of a nonvertical tangent line to the circle (A)
and (x, y) is a point on the tangent line then
(y 0 )2 (x2 − 1) − 2xyy 0 + y 2 − 1 = 0. (C)
(c) Show that the segment of the tangent line (B) on which (x − x0 )/y0 > 0 is
an integral curve of the differential equation
p
0 xy − x2 + y 2 − 1
y = , (D)
x2 − 1
while the segment on which (x − x0 )/y0 < 0 is an integral curve of the
differential equation
p
xy + x2 + y 2 − 1
y0 = . (E)
x2 − 1
(d) Show that the upper and lower semicircles of (A) are also integral curves of
(D) and (E).
(e) Find the equations of two lines through (5,5) tangent to the circle (A), and
find the points of tangency.
(16) (a) Show that the equation of the line tangent to the parabola
x = y2 (A)
at a point (x0 , y0 ) 6= (0, 0) on the parabola is
y0 x
y= + . (B)
2 2y0
(b) Show that if y 0 is the slope of a nonvertical tangent line to the parabola (A)
and (x, y) is a point on the tangent line then
4x2 (y 0 )2 − 4xyy 0 + x = 0. (C)
(c) Show that the segment of the tangent line defined in a) on which x > x0 is
an integral curve of the differential equation
p
y + y2 − x
y0 = , (D)
2x
while the segment on which x < x0 is an integral curve of the differential
equation p
y − y2 − x
y0 = , (E)
2x √
(d) Show that √ the upper and lower halves of the parabola (A), given by y = x
and y = − x for x > 0, are also integral curves of (D) and (E).
(17) Use the results of Exercise 16 to find the equations of two lines tangent to the
parabola x = y 2 and passing through the given point. Also find the points of
tangency. (a) (−5, 2) (b) (−4, 0) (c) (7, 4) (d) (5, −3)
(18) Find a curve y = y(x) through (1,2) such that the tangent to the curve at any
point (x0 , y(x0 )) intersects the x axis at xI = x0 /2.
(19) Find all curves y = y(x) such that the tangent to the curve at any point
(x0 , y(x0 )) intersects the x axis at xI = x30 .
(20) Find all curves y = y(x) such that the tangent to the curve at any point passes
through a given point (x1 , y1 ).
(21) Find a curve y = y(x) through (1, −1) such that the tangent to the curve at any
point (x0 , y(x0 )) intersects the y axis at yI = x30 .
73
(22) Find all curves y = y(x) such that the tangent to the curve at any point
(x0 , y(x0 )) intersects the y axis at yI = x0 .
(23) Find a curve y = y(x) through (0, 2) such that the normal to the curve at any
point (x0 , y(x0 )) intersects the x axis at xI = x0 + 1.
(24) Find a curve y = y(x) through (2, 1) such that the normal to the curve at any
point (x0 , y(x0 )) intersects the y axis at yI = 2y(x0 ).

In Exercises 25–29 find the orthogonal trajectories of the given family of curves.
(25) x2 + 2y 2 = c2
(26) x2 + 4xy + y 2 = c
(27) y = ce2x
2
(28) xyex = c
(29) y = cex /x
(30) Find a curve through (−1, 3) orthogonal to every parabola of the form
y = 1 + cx2
that it intersects. Which of these parabolas does the desired curve intersect?
(31) Show that the orthogonal trajectories of
x2 + 2axy + y 2 = c
satisfy
|y − x|a+1 |y + x|a−1 = k.
(32) If lines L and L1 intersect at (x0 , y0 ) and α is the smallest angle through which
L must be rotated counterclockwise about (x0 , y0 ) to bring it into coincidence
with L1 , we say that α is the angle from L to L1 ; thus, 0 ≤ α < π. If L and L1
are tangents to curves C and C1 , respectively, that intersect at (x0 , y0 ), we say
that C1 intersects C at the angle α. Use the identity
tan A + tan B
tan(A + B) =
1 − tan A tan B
to show that if C and C1 are intersecting integral curves of
f (x, y) + tan α
y 0 = f (x, y) and y0 = (α 6= π/2) ,
1 − f (x, y) tan α
respectively, then C1 intersects C at the angle α.
(33) Use the result of exampleercise 32 to find a family of curves that intersect every
nonvertical line through the origin at the angle α = π/4.
(34) Use the result of exampleercise 32 to find a family of curves that intersect every
circle centered at the origin at a given angle α 6= π/2

Exercises
(1) True or false. Don’t forget to justify your answer.
(a) e−t cos(3t)y 0 = y is a first order, linear, separable ordinary differential equa-
tion.
(b) (x + 2)xex sin y + (x2 ex cos y)y 0 = 0 is NOT an exampleact equation.
(c) If y1 is a solution of y 00 + 4y 0 + 5y = 0, so is 3y1 .
(2) Solve the initial value problem
3
y 0 + 2y = t; y(0) = .
4
74
(3) Let y(t) be the particular solution of the following initial value problem
y 0 = (−y)(2 − y)(y + 4); y(3) = −2
What is lim y(t) ?
t→∞
x2
(4) Solve y 0 =
y(1 + x3 )
0 x2
(5) For y = , write an initial condition so that it has a unique solution.
y(1 + x3 )
x2
(6) For y 0 = , write an initial condition so that it has no solution.
y(1 + x3 )
x2
(7) For y 0 = , write an initial condition so that it has more than one
y(1 + x3 )
solution.
(8) Match the following direction fields with their differential equations.
(I) yy 0 + x = 0
(II) y 0 + 1 − y 2 = 0
(III) x2 y 0 − y = 0
(IV) (y − 1)y 0 + x3 − 4x = 0
(9) Solve t3 y 0 + 4t2 y = e−t ; y(−1) = 1.
dy −xy − y 2
(10) Solve = 2
dx 2x + 5xy
(11) Solve the differential equations:
3
(a) y 0 + 2ty = 0, y(1) = −
e
1 0 −t2
(b) y + y = e
2t
(12) Sketch a rectangle so that the follwoing IVP has a unique solution there.
y 0 cot(t)y 2 (1 + y)−1 , y(π/2) = 2.
(13) Solve (2t2 + 2y)y 0 + (2ty 2 + 2t) = 0
dy 4t + 3y
(14) Find the general solution of =− .
dt 2t + y
(15) Solve y 0 + (1/t)y = 2 sin t; t > 0.
8 + t3
(16) Consider IVP y 0 = ; y(2) = 1. Find a rectangle on which IVP has a
3y − y 2
unique solution.
(17) Draw direction field.

75

Vous aimerez peut-être aussi